Naked Science Forum

Non Life Sciences => Physics, Astronomy & Cosmology => Topic started by: NeT-HeaD on 10/03/2016 19:24:55

Title: Why do photons have the same speed regardless of the velocity of their source?
Post by: NeT-HeaD on 10/03/2016 19:24:55
I learned that light travels at the same given speed no matter if a light source is static in a fixed place or the source is moving towards you in that same space.

What is it that decelarates a photon that is emitted from an object moving toward you with say half the speed of light to come back to exactly the fixed speed of light.

in other words, the photon would leave the lightsource at half the speed of light in order to arive at your position with the exact lightspeed , or am i seeing this wrong?

So 'something' must slow it down otherwise it would arive at one and a half times lightspeed.
What is this 'something'?
Title: Re: Why do photons emmited from a moving source and a static source have same speed
Post by: evan_au on 10/03/2016 20:15:07
The fact that the speed of light in a vacuum is measured the same by all observers is really hard to get your head around!

Einstein showed that when you travel at really high speeds (like half the speed of light, in your example), your measurement of length and your measurement of time changes. He later showed that time distortion also occurs in a gravitational field.

They change in such a way that you always measure the speed of light in your laboratory to be c≈3x108 m/s.

So the light does not change it's speed in the universe; your perception of the universe changes.

See: http://en.wikipedia.org/wiki/Lorentz_transformation

Note that although light does not change its speed, different observers will measure the frequency and energy of the photons to be different.
Title: Re: Why do photons emmited from a moving source and a static source have same speed
Post by: NeT-HeaD on 11/03/2016 02:08:37
hmm.
I'm not any closer to understanding this and get the relativity between spacetime and light.

In my example i was not traveling at any speed . simply observing a photon reachin me from a static fixed point(lets say our sun) and a photon reachin me from a towards me moving object(lets say a meteorite like object reflecting or emmiting a foton towards me while the object travels at half the speed of light)

i cannot see how both photons have the same speed unless the  one emmited from the moving meteorite was somehow slowed down by something.
Anyway , tnx for the answer , you gave me some more background that i didn't have before :)

All in all i could be looking at this all wrong maybe.

A photos is a partical moving in space and time . Maybe the properties of space and time just simply don't alow for faster
travel. or movement. Or maybe it isn't moving at al because it's proppelled towards me but maybe i'm atracting it towards me instead and it really is a matter of perceptual dynamics.

I need more coffee...
Title: Re: Why do photons emmited from a moving source and a static source have same speed
Post by: Colin2B on 11/03/2016 15:47:48
Try not to think of the photons as tiny bullets fired from the atom. Think more of vibrations. If you move your hand up and down on the surface of a pond the vibrations will create waves. The same thing happens with light and sound, but the speed that those vibrations travel depends on the medium they are travelling through, not on the speed of the object that produced the vibrations.
So, for an aircraft on the ground or flying towards you the speed of the sound is entirely dependant on the properties of the air and is the same for a stationary plane or one coming towards you at 500mph. It's not that the sound slows down, it's just that it wasn't moving until the instant it was generated.
However, there is a big difference between sound and light. If an aircraft is stationary and we move towards it we will measure the speed of the sound of its engine to be the speed of sound in air minus our own speed. This is because we are also moving in the medium ie air, so we are moving relative to the air which contains the sound waves.
With light however, there is no medium that the light is moving through, so if you move towards a light source you will still measure the same speed of light as if you we're stationary . People used to think there was such a medium and called it aether.
I'm afraid this is a very simplified explanation, skips a lot of detail and doesn't cover all the issues, but I hope it gives you something to visualise.
Title: Re: Why do photons emmited from a moving source and a static source have same speed
Post by: NeT-HeaD on 11/03/2016 20:44:03
Tnx Colin,

The theories and comparison of light and sound , and the doppler effect when soundsourses(or receivers) move were allready known to me.

I'm simply(well , not simple at all, but as a figure of speech) trying to understand why lightspeed is a fixed given absolute limit when it comes to moving through space.
Nothing can move faster (according to einsteins and other theories) and i want to know what it is that makes everything hold up to this fact.
As you said, in space is nothing that can 'vibrate' (or be modulated) but still a lightsource does do exactly that. Light is indeed a 'movement' in a particular frequency and amplitude but what it is what 'vibrates' and gets 'modulated' to transport the properties of light so it becomes visible at every distance from the emiting source blows my mind. simply cannot figure it out.
I must remind everyone here that i am no scientist or not even a student. i only found this matter interesting all of my life but always got put off by loads of 'shop talk'i cannot grasp.

Tnx for your attempt.
 
 
Title: Re: Why do photons emmited from a moving source and a static source have same speed
Post by: Colin2B on 11/03/2016 23:21:21
I must remind everyone here that i am no scientist or not even a student. i only found this matter interesting all of my life but always got put off by loads of 'shop talk'i cannot grasp.

Tnx for your attempt.
No, I should thank you for trying.
This problem took the best brains hundreds of years to work out what was happening and even now there are gaps in the detail.
I think you come close to the real problem when you ask 'what vibrates'. This is a trap most of us fall into, we are so used to thinking of waves in water or sound waves in air that we think there must be some thing which vibrates. The truth is that it is the magnetic and electric fields which vibrate and they don't need anything to support them, they work anywhere even in a total vacuum.
The area you are tackling is not an easy one, but don't let that put you off trying, you are asking all the right questions.
Title: Re: Why do photons have the same speed regardless of the velocity of their source?
Post by: NeT-HeaD on 20/03/2016 12:49:14
Tnx colin,

It took some time of letting go of this mater to get a 'flash' of insight. I now understand why doppler doesn't apply to lightsourses that move.

It's simple really. The moment a photon leaves its source it starts traveling at lightspeed and because relative to this lightspeed  this source is a point of origin that sits  still  and is actually static because the emmited photons speed is already the maximum and constant. there's is no acceleration. light emmision is instant. and there lies the clue.
i think.
Title: Re: Why do photons have the same speed regardless of the velocity of their source?
Post by: Colin2B on 20/03/2016 13:19:23
It's simple really. The moment a photon leaves its source it starts traveling at lightspeed and because relative to this lightspeed  this source is a point of origin that sits  still  and is actually static because the emmited photons speed is already the maximum and constant. there's is no acceleration. light emmision is instant. and there lies the clue.
i think.
That's pretty much it.

Only complication is that Doppler shift does apply to moving sources. Although the light has the constant speed once emitted, with a moving source the light waves are squished in the direction of motion, so the wavelength shortens, the frequency increases and the colour becomes bluer. Opposite if the source moves away from a receiver.
Title: Re: Why do photons have the same speed regardless of the velocity of their source?
Post by: jeffreyH on 20/03/2016 13:44:40
If we think of the universe as a fixed 3 dimensional grid with each cell having the dimensions (x, y, z) and where x=y=z we then have a universe divided into cubes. Light can move from one of these cubes to another in L amount of time where we have L as representing a direct relationship to the speed of light. Nothing else in the universe can move this fast and so the time taken for anything else to change cubic position has to be less than L. At the very smallest scales this is what happens. Only we cannot directly observe it. To our intuitive senses at our macroscopic scale it doesn't make sense. Like much of quantum mechanics at the microscopic scale.
Title: Re: Why do photons have the same speed regardless of the velocity of their source?
Post by: timey on 20/03/2016 13:52:44
Only complication is that Doppler shift does apply to moving sources. Although the light has the constant speed once emitted, with a moving source the light waves are squished in the direction of motion, so the wavelength shortens, the frequency increases and the colour becomes bluer. Opposite if the source moves away from a receiver.

However...  The Pound Rebka experiment shows that a blueshift, and a redshift of lights wavelength occurs when the emitter, and the receiver of the light source are both held """static""" relative to each other!!!
Title: Re: Why do photons have the same speed regardless of the velocity of their source?
Post by: Colin2B on 20/03/2016 14:52:46
Only complication is that Doppler shift does apply to moving sources. Although the light has the constant speed once emitted, with a moving source the light waves are squished in the direction of motion, so the wavelength shortens, the frequency increases and the colour becomes bluer. Opposite if the source moves away from a receiver.

However...  The Pound Rebka experiment shows that a blueshift, and a redshift of lights wavelength occurs when the emitter, and the receiver of the light source are both held """static""" relative to each other!!!
But I was talking about Doppler shift due to movement.
Title: Re: Why do photons have the same speed regardless of the velocity of their source?
Post by: timey on 20/03/2016 15:21:03
Well yes - of course Colin!

And isn't it interesting that a representation of Doppler shift can perhaps be found in the redshift, blueshift phenomenon between a light source emitter and receiver that are indeed held static relative to each other?
Title: Re: Why do photons have the same speed regardless of the velocity of their source?
Post by: JoeBrown on 20/03/2016 19:22:50
Well yes - of course Colin!

And isn't it interesting that a representation of Doppler shift can perhaps be found in the redshift, blueshift phenomenon between a light source emitter and receiver that are indeed held static relative to each other?

Doppler shift doesn't occur when source/reception are static.  That's not what the experiment showed, nor was it the intention.  They were verifying Einstein / gravity shift.  They used relative doppler motion/shift to cancel the effect of gravity shift.
Title: Re: Why do photons have the same speed regardless of the velocity of their source?
Post by: JoeBrown on 20/03/2016 19:39:07
*I think* the basic premise of light speed in relativity boils down to this:

The act of acceleration and motion affect observers and emitters time dilation.  This time dilation cancels the speed differential to that of light speed.

Tho that doesn't explain why it is what it is, it kinda explains how it works that way.  Tho I'm not an expert on relativity.  I'm still trying to figure out how Albert came up w/that, imagining he was running alongside a light corpuscle...  I've imagined similar and seen nothing of the sort.
Title: Re: Why do photons have the same speed regardless of the velocity of their source?
Post by: timey on 20/03/2016 20:28:56
Well yes - of course Colin!

And isn't it interesting that a representation of Doppler shift can perhaps be found in the redshift, blueshift phenomenon between a light source emitter and receiver that are indeed held static relative to each other?

Doppler shift doesn't occur when source/reception are static.  That's not what the experiment showed, nor was it the intention.  They were verifying Einstein / gravity shift.  They used relative doppler motion/shift to cancel the effect of gravity shift.

Firstly, what an experiments intention is has no bearing on the use of its results.

You are stating that a relative Doppler  motion/shift was used to cancel out gravity shift...

Therefore if the blueshift redshift effects between a light sources emitter and receiver that are held static relative to each other can be cancelled out by relative Doppler motion/shift, then there is indeed a representation of a Doppler shift within the redshift blueshift observations within gravity shift.

P.S.  In answer to your second post:

You are forgetting that the Pound Rebka was initiated with a view to measuring gravitational shift with respect to time dilation considerations.  The gravitational field itself experiences time dilation.  It's not just a case of SR time dilation with regards to moving objects... you know!
Title: Re: Why do photons have the same speed regardless of the velocity of their source?
Post by: JoeBrown on 20/03/2016 22:07:20
Firstly, what an experiments intention is has no bearing on the use of its results.
It wasn't the intention to show doppler shift occurs in static relative motion, nor was it shown.  It showed that gravitational shift matched the predicted (expected) results.

Which was the point.  Doppler shift doesn't occur w/out differences in relative motion. That's how it is understood and it is how it works.

Those are the facts.  I intended to point out your previous statement was interestingly inaccurate.

Quote
You are forgetting that the Pound Rebka was initiated with a view to measuring gravitational shift with respect to time dilation considerations.  The gravitational field itself experiences time dilation.  It's not just a case of SR time dilation with regards to moving objects... you know!

I didn't forget...  Tho I'm still no expert.

I omitted reference to gravity, to provide a simple(r) answer to initial question.  Why over complicate?  Is it prudent to use general when special relativity achieves the desired result?
Title: Re: Why do photons have the same speed regardless of the velocity of their source?
Post by: timey on 20/03/2016 22:51:58
Yup - you are correct.  It wasn't the intention to show doppler shift occurs in static relative motion... But, in that a Doppler shift cancelled out the """relative motion""" of the redshift blueshift observations, it 'is' shown, whether intended or not, that a representation of the Doppler shift is apparent within the gravitational shift of redshift blueshift observations between a light source emitter and receiver that are held """static""" relative to each other...  And this 'is' interesting!

Yup -  The experiment did show that gravitational shift matched the predicted (expected) results with regards to a clock ticking faster in elevation.

Nope - I am not over complicating matters.  Call it GR or SR at your will and want.  Fact is, according to currently held theory, you get time dilation for objects travelling at relativistic speeds, and you get time contraction in a reducing gravitational field.
Title: Re: Why do photons have the same speed regardless of the velocity of their source?
Post by: jeffreyH on 21/03/2016 08:22:12
If it is even meaningful we could have the fastest rate of time equalling 1. In other words 100%. This would have to occur at infinity outside the influence of gravity. Since infinity is not on the number line nothing can reach it. This is exactly why there is no fixed background. Time will never run at full speed and neither will it completely stop as both states are absolutes that can never be reached.
Title: Re: Why do photons have the same speed regardless of the velocity of their source?
Post by: Colin2B on 21/03/2016 08:49:15
Time will never run at full speed and neither will it completely stop as both states are absolutes that can never be reached.
What happens for the free falling observer Jeff? I assume time runs at 100%. As she falls (pushed off the tower by those who disagree with her theory) she looks up and sees the source receding- red shifted, looking down she realises the detector is moving up against the light waves - blue shifted. Her view is that there is no gravity, just acceleration and all  effects at top and bottom of tower are due to doppler shift.
Title: Re: Why do photons have the same speed regardless of the velocity of their source?
Post by: puppypower on 21/03/2016 11:45:19
The simplest explanation for why photons always move at the same speed, regardless of reference, is the speed of light is the ground state of the universe. This can be easily inferred. In our universe, there is a net conversion of mass to energy; fusion, and not a net conversion of energy to mass. This reflects that the net direction of potential in the universe, is the lowering mass potential toward an energy based ground state. All the forces of nature give off energy, as one of their lower potential products.

As a visual analogy, say we assume sea level is the ground state for the surface of the earth, since all the water goes there. It does not matter where you begin as your reference; hill, mountain or cloud. They will use the same ground state. There may be different potentials with this ground state; reflected in the wavelength of the energy, but all water goes to sea level.

If you look at gravity, gravity will cause mass to clump causing local space-time to contract. If we contract space-time to the limit of a point-instant, we will get the speed of light reference. Gravity is a reflection of space-time lowering potential back to the speed of light reference; toward the point-instant. This point-instant limit; speed of light reference, is achieved by the black hole.

If you were traveling at the speed of light, and looked out at the universe, the universe would appear like a point-instant.  That being said, the only wavelength of energy/photons you would be able to see would be infinite wavelength energy. The reason is anything with a smaller wavelength than infinite, will have a wavelength that is a fraction of a point in size. This is not mathematically possible by definition. A point is a small as you can get by definition. This implies all energy, with shorter wavelength than infinite, will need to lower potential by gaining wavelength, to reach the ground state; all energy will need to red shift.

All roads lead to the speed of light ground state in their own ways. This is why all references, will see and make use of the exact same ground state.

The advantage of using the C ground state, instead of an inertial zero state, is you can start the universe earlier than with the singularity of the big bang. The reason is the singularity sets a potential with the C ground state. The singularity defines a state of highest potential that needs to expand. It needs to expand to deal with the energy and needs to clump to deal with the mass. It also needs forces to deal with other states of matter.

I won't get into this, since such a discussion diverges to much from the topic at hand by starting earlier than the inertial time=0; state of highest potential.
Title: Re: Why do photons have the same speed regardless of the velocity of their source?
Post by: timey on 21/03/2016 17:55:23
What happens for the free falling observer Jeff? I assume time runs at 100%. As she falls (pushed off the tower by those who disagree with her theory) she looks up and sees the source receding- red shifted, looking down she realises the detector is moving up against the light waves - blue shifted. Her view is that there is no gravity, just acceleration and all  effects at top and bottom of tower are due to doppler shift.

Thanks for the push Colin!

Whaaaaargh....  I'm faaaaallliiiing!

...at a rate of 9.8m/s2 acceleration...
As I look behind me, I can see blue shifted light rushing towards me, but from the point that it passes me I cannot see it redshift towards the earth from my position.  My position in the field is changing, but as the light is travelling at the speed of light, it's position in the field changes faster than mine.

Alternatively, the second time you pushed me off the tower, (clearly I hadn't yet learned my lesson ;) )...  light is being sent way from Earth.  I look up behind me as I fall, I cannot see the light redshift away behind me... but looking downwards, up till the point that the light passes my position in the field, I can see it redshift towards me.

Now, then... having been pushed off the tower twice, I think I can be forgiven for a bit of wonky thinking, or could it perhaps be that I have become en-light-ened?

My observations when falling where as such:
Firstly:
A Doppler shift was used to cancel out the redshift and blueshift effects, thereby proving that these effects of redshift and blueshift exist as a result of the changes in the gravitational field.
That as a direct result of the observation above in relation to the light source emitter and receiver being held static relative to each other, there is proven a representation of Doppler shift within the relative motion of light via changes in the gravity field.
That both of the above considered in relation to the constant speed of light and the 'constant distance' between ground and top of tower of the experiment, shows that something is a little amiss.  Light should not have any such Doppler shift relative motion properties over a constant distance, travelling at a constant speed!
And... that taking into account that the light will be travelling at the speed of light under the currently thought remit of the minuscule time drifts occurring within the changes in the gravitational field - 'does not' in fact describe the relative motion of a Doppler shift!

Secondly:
That I was falling at a rate of 9.8m/s2.
That the first time I was pushed off the tower, my mobile phone fell out of my pocket... and in that I arranged my mass as to be as lacking in air resistance as my phones mass, I thought I noticed my phone and I arrived on the ground at the same time.
That the second time you pushed me off the tower Colin, with great presence of mind, to say so, I picked your pocket of your phone and repeated the experiment.  Your phone and my phone, (both smashed to smithereens now of course) did not weigh the same, in weight or worth.  Mine was heavier, yours more expensive!  But, again, I noticed that your phone and I arrived on the ground at the same time.

Why does the light not accelerate at 9.8m/s2?  Answer:  Because the speed of light is constant.
Are we expecting that the light will have taken a couple of a billionth of a second longer time to travel the distance on account of the gravity field slowing time down?  Answer:  Yes we do.
We have attributed light with mass equivalent to its energy.  We state that the gravitational field robs the light of energy when leaving a gravitational field, ie: redshift.  So... what exactly is replenishing lights energy when we observe a blueshift?

The energy of mine and the phones falling is easily explained due to potential energy, via the fact of my mass and my position, or changing position, within the gravitational field.

The fact of the lights increase in energy found in its falling towards a gravitational field is described by adding relativistic mass.  But, by this remit, as my mass, or the mass of the phones falls towards earth it must also increase.  Again... we come back to the question as to why lights mass does not 'accelerate' at 9.8m/s2, like mine does?

Looking at the energy of my mass, we can see that as I fall, the potential energy that my mass experiences is reducing as I fall into the gravitational field.  Therefore, the energy that light experiences as a result of the adding of relativistic mass, also will 'reduce' as it falls towards earth.  But light energy does not reduce as it gets closer to a gravitational field.  This constitutes a breach of logic, unless it is the increase in inherent mass energy as the relativistic mass is increased that describes the increase in energy.  It should be noted, that adding relativistic mass to light as its energy 'increases' would not subject the light to any more, or any less than an acceleration of 9.8m/s2!
Yet light is not subject to any 'further' acceleration of its own speed under the influence of a gravitational field.

However... I observed that a Doppler shift cancelled out the redshift blueshift observations from top and bottom of tower perspectives.  These perspectives of the light source emitter and receiver having been held static relative to each other!!!
This alerts me to these 5 considerations:
a) That there is a relative motion in light representing a Doppler shift that does not incorporate any relative motion of distance.
b) That on the basis of there being a relative motion in light representing a Doppler shift that did not incorporate any relative motion of distance, that redshift considerations are not necessarily indicative of an expanding universe.
c) That the relative motion of Doppler shift found in light that is apparent over a 'constant distance' does not add up to gravitational shift, plus time drift consideration, plus the aspect of potential energy reduction in light caused by decreasing distance between its mass and the mass of the earth as it enters further into the gravitational field.  (or at least not without rendering distance as a variable.)
d) That if I were to look upon an increasing gravitational field increasing the rate of time... that the fact of a Doppler shift cancelling out redshift blueshift frequencies indicates that light, travelling at the speed of light, is accelerated towards a gravitational field via an increase in the rate of time that directly relates to the increase in a gravitational field via a representation of Doppler shift.  That light is simply getting its energy directly from the gravitational field, it's frequency is indicative of the rate of time, and it's additions or reductions in wavelength are not 'distance' related, but 'time' related.
e) That perhaps there is no acceleration of gravity, perhaps it is instead only an acceleration of time.

So... what is happening when I am falling?  The difference between light and I being that light has no mass... and, more importantly, it is in fact proven that my clock will run faster in a reduced gravitational field, not slower.  As me and my clock fall, it's rate of time will register ever so slightly slower.
Ok, my mass is subject to gravity potential, but if light 'gets' its energy from a gravitational field, do I also 'get' energy from the gravitational field?  And if I do, then does this effect the status of my e=mc2 inherent mass energy?  ie: having rid ourselves of the idea of relativistic mass for light, does my mass 'change' in a gravitational field, changing my inherent mass energy?

If I state my inherent mass energy as constant, and add on the aspect of potential energy, plus gravitational field energy... given that this describes an 'energy level' that will produce frequency and wave length, does my atomic clocks caesium atoms inherent mass energy, plus the potential energy, plus the gravitational field energy that it is experiencing at elevation, describe the fact of the frequency of its cycles being increased?
Can the aspect of the kinetic energy of a moving body of 'mass' be deducted from the additions of inherent mass energy with potential energy and gravitational field energy, to then describe a slowing of time for both co-moving, and more obviously, non co-moving objects?

And finally...  If so, then could the phenomenon of time itself be 'energy' related?
Title: Re: Why do photons have the same speed regardless of the velocity of their source?
Post by: alancalverd on 21/03/2016 19:00:29
Keep it simple.

Doppler shift is due to relative speed of source and receiver. Gravitational shift is due to relative gravitational potential of source and receiver. They are not the same phenomenon but have the same effect on wavelength, so you can use one to measure the other.

As for the original question, there is no "why". It just happens that an awful lot of observations become interconsistent if you assume that c is constant for all observers, and as far as we can ascertain, it is. The task of teaching physics is to explain why everything else is as it is, given only that c is constant. 
Title: Re: Why do photons have the same speed regardless of the velocity of their source?
Post by: timey on 21/03/2016 19:55:39
Keeping it simple...

Doppler shift is due to relative speed of source and receiver.  But... if there is no relative speed between source and receiver, then any representation of Doppler shift 'must' be due to relative motion within the propagation of light within a gravitational field.

Keeping it simple...

Gravitational shift is due to relative gravitational potential of source and receiver. Doppler shift and gravitational shift are not the same phenomenon but have the same effect on wavelength, so you can use one to measure the other...  But... If there is no relative expansion of distance via a gravitational shift, there is 'again' no relative motion for a Doppler shift to occur in.

Keeping it simple...

How can lights frequency be cancelled out by a Doppler shift between a light source emitter and a light source receiver that are both held static of motion relative to each other - and consist of a 'constant' distance apart?
Title: Re: Why do photons have the same speed regardless of the velocity of their source?
Post by: evan_au on 21/03/2016 20:15:43
Quote
How can lights frequency be cancelled out by a Doppler shift between a light source emitter and a light source receiver that are both held static of motion relative to each other - and consist of a 'constant' distance apart?
If two objects are not moving relative to each other, their Doppler shift is zero.
The only way for Doppler shift to equal gravitational shift is if the gravitational shift is also zero.
Gravitational shift is zero if the two observers are at the same gravitational potential, eg both standing on the surface of the Earth.

Quote
Doppler shift and gravitational shift are not the same phenomenon but have the same effect on wavelength, so you can use one to measure the other
This is true in one direction only.

If you observe the moving light source from multiple directions (or watch it as it passes you), you will see that gravitational shift is cancelled in one direction, but not others.

The reason is that the Doppler effect can produce red and blue shift. However, gravitational frequency shift produces the same shift whether the object is moving towards you or away from you. This is called Relativistic Beaming (http://en.wikipedia.org/wiki/Relativistic_beaming).
Title: Re: Why do photons have the same speed regardless of the velocity of their source?
Post by: timey on 21/03/2016 20:36:02
Evan - my point is that within the Pound Rebka, a Doppler shift was used to cancel out the frequency of redshift from bottom of tower to top of tower.  And again to cancel out the frequency of blueshift from top of tower to bottom.

There was no relative speed between the top of tower and bottom of tower in either instance.  There was also no relative expansion or contraction of the distance between top of tower and bottom of tower in either instance.

Yet - a Doppler shift cancelled the blueshift redshift frequencies between the 2 'static in motion' and 'constant in distance' locations... How did it do this?
Title: Re: Why do photons have the same speed regardless of the velocity of their source?
Post by: JoeBrown on 21/03/2016 22:23:51
Yet - a Doppler shift cancelled the blueshift redshift frequencies between the 2 'static in motion' and 'constant in distance' locations... How did it do this?

They gravitational shifted the height of the tower.  -- errr.  Trying, to be  a wisenhymer.

They moved the light source and/or receiver, which produced doppler shift to cancel gravitational shift.  I don't believe they had to change the height of the tower to accomplish the feat.
Title: Re: Why do photons have the same speed regardless of the velocity of their source?
Post by: timey on 21/03/2016 22:39:20
Joe - I can't believe I am actually dignifying your post of absolute and total ignorance with an answer... But here goes...

They did not 'move' the light source or the receiver.  They did not extend or shorten the 'distance' from the bottom of tower to the top of tower.

Why don't you read up on the subject 'before' commenting?

https://en.m.wikipedia.org/wiki/Pound–Rebka_experiment
Title: Re: Why do photons have the same speed regardless of the velocity of their source?
Post by: Colin2B on 21/03/2016 23:22:23
.

There was no relative speed between the top of tower and bottom of tower in either instance. 
But there was, the source was mounted on a loudspeaker cone so it moved towards and then away from the receiver.
Title: Re: Why do photons have the same speed regardless of the velocity of their source?
Post by: timey on 21/03/2016 23:49:57

Colin - they created a Doppler shift by vibrating the speaker at frequencies between 10 and 50 hertz to cancel out the redshift  blueshift frequencies.  The distance between the speaker and the receiver remained at 22.5 metres.  Therefore the distance between the emitter and receiver remained at 22.5 metres and the distance between the top of tower and bottom of tower remained at 22.5 metres.  The gamma rays travelled 22.5 metres 'only'.

When the Doppler shift canceled out the gravitational blueshift, the receiving sample absorbed gamma rays and the number of gamma rays detected by the scintillation counter dropped accordingly. The variation in absorption could be correlated with the phase of the speaker vibration, hence with the speed of the emitting sample and therefore the Doppler shift.

A Doppler shift in light was identified in this experiment whereas there was no relative motion between the emitter of the gamma rays and the receiver, and no contraction or expansion between the top of tower and bottom of tower.

How did it do this?

Clearly redshift is thought indicative of an expanding universe, whereby the light emitter and receiver are experiencing an expansion of distance between themselves, at an accelerated rate.

The Pound Rebka indicates that redshift from bottom of tower to top of tower shows no such thing!  The light source is not accelerating away. There is no expansion of distance.
Title: Re: Why do photons have the same speed regardless of the velocity of their source?
Post by: timey on 22/03/2016 00:10:32
Therefore the only explanation for any relative motion is within the movement of the light!  But this is not possible, because the speed of light is constant.  The only other aspect in this instance that would affect relative motion is 'time', and I'm sorry, but a Doppler shift of time in accordance with redshift blueshift frequencies is entirely contrary to currently held theory!

This being my point...
Title: Re: Why do photons have the same speed regardless of the velocity of their source?
Post by: JoeBrown on 22/03/2016 00:11:25
Perhaps they selectively applied cosmological shift to the tower?

The wiki article you linked to states:
Quote
By vibrating the speaker cone the gamma ray source moved with varying speed, thus creating varying Doppler shifts. When the Doppler shift canceled out the gravitational blueshift, the receiving sample absorbed gamma rays and the number of gamma rays detected by the scintillation counter dropped accordingly. The variation in absorption could be correlated with the phase of the speaker vibration, hence with the speed of the emitting sample and therefore the Doppler shift.

The vibration of the speaker caused a change in relative motion.  They weren't quite statically separated.

However, as you pointed out, shift has nothing to do with the subject.  So why are we beating this poor horse? To give it some cosmological shift?
Title: Re: Why do photons have the same speed regardless of the velocity of their source?
Post by: timey on 22/03/2016 00:36:30
Yup - you are definitely a little on the dense side to be sure!

The definitive statement here being "When the Doppler shift canceled out the gravitational blueshift, the receiving sample absorbed gamma rays"

For the gravitational blueshift to be cancelled out by the phase of the speaker, the blueshift of the light would have to match the phase of the speaker.

Does this compute?
Title: Re: Why do photons have the same speed regardless of the velocity of their source?
Post by: JoeBrown on 22/03/2016 01:42:42
Timey, I must be dense.

When a speaker vibrates, it is considered motion.  Motion causes doppler shift.

You indicated that doppler shift occurred without motion, when motion was clearly stated to be involved.

I have no idea what's supposed to compute. I calculate you have completely read past the word "motion"

--
I've read these sentences, completely ignoring the word motion.  Makes me kinda sea sick, like your argument, whatever that might be.
Title: Re: Why do photons have the same speed regardless of the velocity of their source?
Post by: timey on 22/03/2016 02:31:18
Dense was perhaps too mild a term...

The speaker phase consists of the motion of a Doppler shift.  The light matches the motion of a Doppler shift, proven by the phase of the speaker cancelling out the frequency of the light.  However, the light emitter and the light receiver are NOT in motion.  Neither does the distance between the light emitter and the receiver contract or expand.  Therefore the Doppler shift 'motion' identified by the experiment is due to a Doppler shift in the light itself.

On the basis that the premiss of the experiment has clearly gone over your head, never mind the point I'm making.  Legs on you, an egg wouldn't crack if it fell out of your arse!

I look forward to a proper physicists comments...
Title: Re: Why do photons have the same speed regardless of the velocity of their source?
Post by: JoeBrown on 22/03/2016 04:35:17
I look forward to a proper physicists comments...

I fear you are incorrect.  You misunderstand what others comprehend, then argue a misguided point.  That does not make for "proper" comment of physics or anything in general.  Your posts are intentionally rude and ignorantly inappropriate.

I'm dense for trying to explain.  For that I apologize.

--

If you truly wish to prove your erroneous point:

Quote
the gamma ray source moved

Kindly explain how that phrase does not equate to motion.



Perhaps it isn't the word "moved" that's causing the issue.

The gamma ray source, is the light source that moved.
Title: Re: Why do photons have the same speed regardless of the velocity of their source?
Post by: alancalverd on 22/03/2016 07:51:52
Colin - they created a Doppler shift by vibrating the speaker at frequencies between 10 and 50 hertz to cancel out the redshift  blueshift frequencies.  The distance between the speaker and the receiver remained at 22.5 metres.

Either the speaker was vibrating, in which case the distance between source and detector was varying, or it wasn't vibrating and the distance was constant.

Is that enough "proper physics"?
Title: Re: Why do photons have the same speed regardless of the velocity of their source?
Post by: Colin2B on 22/03/2016 09:09:44
Colin - they created a Doppler shift by vibrating the speaker at frequencies between 10 and 50 hertz to cancel out the redshift  blueshift frequencies.  The distance between the speaker and the receiver remained at 22.5 metres.

Either the speaker was vibrating, in which case the distance between source and detector was varying, or it wasn't vibrating and the distance was constant.

Is that enough "proper physics"?
Thanks Alan, my point exactly.
Title: Re: Why do photons have the same speed regardless of the velocity of their source?
Post by: timey on 22/03/2016 10:21:44
Is that enough "proper physics"?

Oh fer goodness sake!!!

My answer:  No, not really!

Yes - the Doppler shift is created by the vibration of the speaker...but the speaker isn't creating the gravitational shift in the light that the vibration of the speaker cancels out.  The speaker cone fabric is moving back and forth, as speakers do, to create vibration, but neither the light source or the receiver are in relative motion with respect to each other, such as a natural light source and receiver.  The distance between the light source and the receiver, apart from this vibration of the speaker is 'static'.

What the experiment shows is that there is a Doppler shift in the motion of light that is not associated with an expanding distance, or an acceleration of the speed of travel of the light source.

How can there be 'relative motion' in light over a distance that is constant when the speed of light is constant?
Title: Re: Why do photons have the same speed regardless of the velocity of their source?
Post by: timey on 22/03/2016 10:35:26
I fear you are incorrect.  You misunderstand what others comprehend, then argue a misguided point.  That does not make for "proper" comment of physics or anything in general.  Your posts are intentionally rude and ignorantly inappropriate.

Actually I am not incorrect.  And I am not being rude.  If you want to accuse someone of being a weisenhiemer, and make idiotic comments about cosmological shift, and intimate that the tower itself suffered a gravitational shift, without actually attempting to understand what the poster is in fact talking about, then you are fair game as far as I'm concerned.  Short legs being an apt description!
Title: Re: Why do photons have the same speed regardless of the velocity of their source?
Post by: McQueen on 22/03/2016 11:07:22
Here is the quote on the location of the gamma ray source:

  A solid sample containing iron (57Fe) emitting gamma rays was placed in the center of a loudspeaker cone which was placed near the roof of the building. Another sample containing 57Fe was placed in the basement.

Since the gamma ray source was placed in the centre of the loudspeaker cone, I assume that the gamma ray source was moving up and down with the vibration of the speaker, causing the distance over which the light had to travel to vary. But in which case where does the doppler shift come in ? In order to establish validity (i.e., to prove that the gravitational blue shift would act as was supposed resulting in non-absorption of the gamma ray) shouldn't a test have been carried out without the speaker, or is that taken for granted ?
Title: Re: Why do photons have the same speed regardless of the velocity of their source?
Post by: puppypower on 22/03/2016 11:37:04
Consider, as an analogy for a photon, a boat traveling on the water. The boat is analogous to the particle, while the wake created by the boat is analogous to the wave aspect of the photon. If we had two tunnels under the bridge, the boat; particle, can only go through one tunnel at a time, while the wake, if spread out, can go through both tunnels.

The boat needs a constant supply of energy to keep moving at velocity V. This energy is needed to overcome friction between the boat and water. If the boat cuts the engines, the particle will slow, coast and then stop, while the wake will decay and finally stop. The speed of light is needed to maintain the wake/wave of the photon particles.

The question becomes how does a photon move through the medium of space-time and not slow down or speed up? What is the source of the propulsion energy?

If we assume the speed of light is the ground state of the universe, this implies all inertial references will be at higher potential. In this case, the constant speed of light would be connected to photons being constantly induced into the ground state ay C. The photons are constantly induced to higher potential; less than C by inertial. They cyclically drop back to the ground state; speed of light, while the energy differential is given off; motor that generates the wake/wave. 

One analogous way to look at this is connected to a house with a variety of electrical circuits, all using the same ground; earth. The ground is common to all the circuits and is at lower potential than any voltage/current configuration we may use. The potential difference between the inertial states, and the ground, is the energy potential used to drive the particle, which creates an analogous wake in space-time. This wake can become modified with the medium of space-time; red and blue shift, but the ground never changes, so the particles are always moving at C.
Title: Re: Why do photons have the same speed regardless of the velocity of their source?
Post by: marzosia2 on 22/03/2016 11:43:53
MOD EDIT: please keep your replies clear ,concise and relevant. Large, multiple posts of diagrams and no explanation will be treated as spam and removed.
Title: Re: Why do photons have the same speed regardless of the velocity of their source?
Post by: timey on 22/03/2016 12:53:18
Consider, as an analogy for a photon, a boat traveling on the water. The boat is analogous to the particle, while the wake created by the boat is analogous to the wave aspect of the photon. If we had two tunnels under the bridge, the boat; particle, can only go through one tunnel at a time, while the wake, if spread out, can go through both tunnels.

The boat needs a constant supply of energy to keep moving at velocity V. This energy is needed to overcome friction between the boat and water. If the boat cuts the engines, the particle will slow, coast and then stop, while the wake will decay and finally stop. The speed of light is needed to maintain the wake/wave of the photon particles.

The question becomes how does a photon move through the medium of space-time and not slow down or speed up? What is the source of the propulsion energy?

If we assume the speed of light is the ground state of the universe, this implies all inertial references will be at higher potential. In this case, the constant speed of light would be connected to photons being constantly induced into the ground state ay C. The photons are constantly induced to higher potential; less than C by inertial. They cyclically drop back to the ground state; speed of light, while the energy differential is given off; motor that generates the wake/wave. 

One analogous way to look at this is connected to a house with a variety of electrical circuits, all using the same ground; earth. The ground is common to all the circuits and is at lower potential than any voltage/current configuration we may use. The potential difference between the inertial states, and the ground, is the energy potential used to drive the particle, which creates an analogous wake in space-time. This wake can become modified with the medium of space-time; red and blue shift, but the ground never changes, so the particles are always moving at C.

Fair enough PuppyPower... BUT... in Pound Rebka """Where"" in space time does this wave LENGTH exist if the distance between light source and receiver is held 'static', with respect to each other, on the basis that the oscillations of the speaker are creating a Doppler shift that cancels out the gravitational shift of the light?
Title: Re: Why do photons have the same speed regardless of the velocity of their source?
Post by: jeffreyH on 22/03/2016 14:06:03
Maybe a little mathematics would clear things up... or maybe not.
Title: Re: Why do photons have the same speed regardless of the velocity of their source?
Post by: timey on 22/03/2016 14:22:55
https://en.m.wikipedia.org/wiki/Doppler_effect

Quoted from link above:
"For waves that propagate in a medium, such as sound waves, the velocity of the observer and of the source are relative to the medium in which the waves are transmitted. The total Doppler effect may therefore result from motion of the source, motion of the observer, or motion of the medium. Each of these effects is analyzed separately. For waves which do not require a medium, such as light or gravity in general relativity, only the relative difference in velocity between the observer and the source needs to be considered."

???
Title: Re: Why do photons have the same speed regardless of the velocity of their source?
Post by: PmbNEP on 22/03/2016 15:18:54
Quote from: NeT-HeaD
I learned that light travels at the same given speed no matter if a light source is static in a fixed place or the source is moving towards you in that same space.

What is it that decelarates a photon that is emitted from an object moving toward you with say half the speed of light to come back to exactly the fixed speed of light.
That's called the invariance of the speed of light. It's the second postulate of special relativity. It's referred to as a postulate because nobody knows why the speed of light is independent of the speed of source.

However you can think of the reason as due to the properties of spacetime. When you change from one frame of reference, e.g. the frame in which the source is at rest, to one in which the source is moving then you're changing to a frame where, as determined from the rest frame, distances parallel to the motion of the source are shorter and where time intervals are shorter, both in exact proportions so that the speed of light has the same speed in all inertial frames.
Title: Re: Why do photons have the same speed regardless of the velocity of their source?
Post by: timey on 22/03/2016 15:30:54
That's called the invariance of the speed of light. It's the second postulate of special relativity. It's referred to as a postulate because nobody knows why the speed of light is independent of the speed of source.

However you can think of the reason as due to the properties of spacetime. When you change from one frame of reference, e.g. the frame in which the source is at rest, to one in which the source is moving then you're changing to a frame where, as determined from the rest frame, distances parallel to the motion of the source are shorter and where time intervals are shorter, both in exact proportions so that the speed of light has the same speed in all inertial frames.

Very good Pmb!!!

Now could you please tell me how, within the Pound Rebka experiment, a Doppler shift can be identified within the gravitational shift of light when there is no relative motion between the light source and the receiver other than the oscillations of the speaker cone... these oscillations of the speaker cone being the method of measuring the fact of the Doppler shift within the gravitational shift of light via cancellation of redshift blueshift frequencies... ???
Title: Re: Why do photons have the same speed regardless of the velocity of their source?
Post by: JoeBrown on 22/03/2016 17:40:21
Now could you please tell me how, within the Pound Rebka experiment, a Doppler shift can be identified within the gravitational shift of light when there is no relative motion between the light source and the receiver other than the oscillations of the speaker cone... these oscillations of the speaker cone being the method of measuring the fact of the Doppler shift within the gravitational shift of light via cancellation of redshift blueshift frequencies... ???

The Pound Rebka experiment used MOTION produced by a speaker cone.  The gamma ray light source was mounted on a MOVING speaker cone, which induced doppler shift.

The argument that doppler shift occurred without motion is flawed.  There was movement.  It didn't require a lot of movement.  Since the distance was in nearly constant flux.  They recorded the phase of the speaker movement to account for doppler shift.

--

Now for my timely wizenhimer remark:  DOH
Title: Re: Why do photons have the same speed regardless of the velocity of their source?
Post by: NeT-HeaD on 22/03/2016 18:18:56
Guy's
All your chatter about dopler dazzles me but . . .
it seems to me that only aplies to interaction between fotons emmited in a row one after another when emmited from moving source or observed by moving source. i wanted to keep the question simple by observing just the ONE photon and its behaviour when emmited form a source.

But hey. it's still very enlightning to follow your guys arguements :)
.
Title: Re: Why do photons have the same speed regardless of the velocity of their source?
Post by: timey on 22/03/2016 18:35:03
The Pound Rebka experiment used MOTION produced by a speaker cone.  The gamma ray light source was mounted on a MOVING speaker cone, which induced doppler shift.

The argument that doppler shift occurred without motion is flawed.  There was movement.  It didn't require a lot of movement.  Since the distance was in nearly constant flux.  They recorded the phase of the speaker movement to account for doppler shift.

--

Now for my timely wizenhimer remark:  DOH

Joe - Again you entirely miss even the 'intended' purpose of the experiment itself.

For the last time -  A Doppler shift was created by mounting the light source in the centre of the speaker cone.  It is sooo obvious that the gamma ray source will move as per the vibration of the speaker that I thought it unnecessary of mention.  It's the entire premiss of the experiment!  They created the measuring Doppler effect using hertz and the subsequent vibration of the speaker in order to cancel out the blueshift redshift frequencies of the light undergoing gravitational shift.

Now then - listen very carefully...  For a Doppler shift in vibration to cancel out the frequencies of blueshift redshift, the frequencies of blueshift redshift must resemble exactly the vibration of the Doppler shift.  Therefore the 'motion' you insist is occurring for the light source, this motion 'actually being' the motion of the speaker, is resembled exactly 'in' the gravitational shift.  Note:  The gravitational shift distance in this experiment is 'static'.  There is no relative motion between the 'distance' of the gravitational shift and observation 'with respect' to the gravitational field, and therefore no """obvious"""" relative motion for a Doppler shift for light in this experiment in the gravitational field.

Redshift is synonymous with expanding distances, and blueshift with contracting distances, as per space time considerations!!!  Yet, as per Pound Rebka, we see a Doppler shift in the gravitational shift of light, measured by a vibration, caused by motion of a speaker receiving between 10 and 50 hertz, over a constant distance in the gravitational field between a light source and receiver that are experiencing no relative motion between themselves, other than the forward and 'backward' motion of a speakers vibration acting upon the light source.

How can there be a Doppler shift in the gravitational shift of light over a constant distance when the speed of light is constant?  What exactly is causing the 'motion' of this Doppler shift found within the gravitational shift of light in this instance?

P.S Net-Head, I'm sorry if we have digressed.  I'm not entirely sure what your interest is in one photon, but I'm pretty sure knowing about what happens to light in general can only help :)
Title: Re: Why do photons have the same speed regardless of the velocity of their source?
Post by: jeffreyH on 22/03/2016 18:46:49
The Pound Rebka experiment used MOTION produced by a speaker cone.  The gamma ray light source was mounted on a MOVING speaker cone, which induced doppler shift.

The argument that doppler shift occurred without motion is flawed.  There was movement.  It didn't require a lot of movement.  Since the distance was in nearly constant flux.  They recorded the phase of the speaker movement to account for doppler shift.

--

Now for my timely wizenhimer remark:  DOH

Joe - Again you entirely miss even the 'intended' purpose of the experiment itself.

For the last time -  A Doppler shift was created by mounting the light source in the centre of the speaker cone.  It is sooo obvious that the gamma ray source will move as per the vibration of the speaker that I thought it unnecessary of mention.  It's the entire premiss of the experiment!  They created the measuring Doppler effect using hertz and the subsequent vibration of the speaker in order to cancel out the blueshift redshift frequencies of the light undergoing gravitational shift.

Now then - listen very carefully...  For a Doppler shift in vibration to cancel out the frequencies of blueshift redshift, the frequencies of blueshift redshift must resemble exactly the vibration of the Doppler shift.  Therefore the 'motion' you insist is occurring for the light source, this motion 'actually being' the motion of the speaker, is resembled exactly 'in' the gravitational shift.  Note:  The gravitational shift distance in this experiment is 'static'.  There is no relative motion between the 'distance' of the gravitational shift and observation 'with respect' to the gravitational field, and therefore no """obvious"""" relative motion for a Doppler shift for light in this experiment in the gravitational field.

Redshift is synonymous with expanding distances, and blueshift with contracting distances, as per space time considerations!!!  Yet, as per Pound Rebka, we see a Doppler shift in the gravitational shift of light, measured by a vibration, caused by motion of a speaker receiving between 10 and 50 hertz, over a constant distance in the gravitational field between a light source and receiver that are experiencing no relative motion between themselves, other than the forward and 'backward' motion of a speakers vibration acting upon the light source.

How can there be a Doppler shift in the gravitational shift of light over a constant distance when the speed of light is constant?  What exactly is causing the 'motion' of this Doppler shift found within the gravitational shift of light in this instance?

P.S Net-Head, I'm sorry if we have digressed.  I'm not entirely sure what your interest is in one photon, but I'm pretty sure knowing about what happens to light in general can only help :)

Now express that in terms even the most pompous can understand. At the moment it is simply unintelligible.
Title: Re: Why do photons have the same speed regardless of the velocity of their source?
Post by: timey on 22/03/2016 19:09:21
Now express that in terms even the most pompous can understand. At the moment it is simply unintelligible.

Jeff - lol!

And for the pompous:

For a Doppler shift, caused by vibration, to cancel out the frequencies of gravitational blueshift redshift, the frequencies of blueshift redshift must resemble exactly the vibration of the measuring Doppler shift.

The measuring vibration of the speaker, ie: the phase, is 'just measuring' the same effect in the gravitational shift of light.  Except, and this is the important bit, oh pompous one ;), unlike the speaker cone, the gravitational field is NOT in motion.

Now try relating that back to Hubble's law!
Title: Re: Why do photons have the same speed regardless of the velocity of their source?
Post by: JoeBrown on 22/03/2016 19:42:10
IMO, Hubble's law or constant or parameter is flawed because it simply does not account for gravitational shift.
Title: Re: Why do photons have the same speed regardless of the velocity of their source?
Post by: timey on 22/03/2016 20:04:57
Joe - As far as I understand, and I am prepared to have my understanding called into question, redshift is thought to be indicative of an expansion between bodies of mass, ie: light source and receiver.  The Doppler shift of redshift is indeed an 'increase' in wave'length'.  Unfortunately GR refuses to provide an absolute reference frame for the universe, due to inconsistencies in measurement under the remit of the speed of light being constant.
IMO, the Pound Rebka proves an absolute reference frame within a gravitational field via redshift blueshift frequencies.  BUT... """Warning""" this requires a rethink on the nature of the phenomenon of time, and it's time dilation, contraction tendencies within the gravitational field.
IMO...the Pound Rebka proves inconsistencies in currently held physics theory.  A Doppler shift of light cannot occur in a gravitational field, unless there is relative motion of the observer, the source, or the field.
If you have got your head around the fact that the motion of the speaker creating a Doppler shift to counter and therefore measure a Doppler shift that is already and naturally occurring in the gravitational field, I'll feel like I've got somewhere in our discussion.
Title: Re: Why do photons have the same speed regardless of the velocity of their source?
Post by: JoeBrown on 22/03/2016 20:28:03
Doppler and gravitational shift are two different phenomena we understand cause shift in frequency of light.

Doppler is pretty well understood because it's effects can be heard in sound waves, seen in light waves and works pretty good in radar.

Gravitational shift is a lot harder to detect, which was the purpose of the "Pound Rebka experiment."

They first measured the shift in light between statically separated source and receivers. Took measurements from top and bottom and noted that the shifts were equal and opposite, as predicted by Einstein / gravity shift.

They further metered the change in shift by comparing it to doppler shift produced by the motion of a speaker.  When they compared the phase (of the speaker motion) that canceled the opposing shifts as predicted, the experiment concluded gravity shift is in agreement with the prediction of GR.

GR purposefully does not provide an absolute reference because its premise is that everything is relative, tho the speed of light is about the closest it comes to an absolute frame of reference.
Title: Re: Why do photons have the same speed regardless of the velocity of their source?
Post by: timey on 22/03/2016 20:55:16
Yawn...

Joe - I too can read the Wiki links and a lot more besides, and have been doing so consistently for over 7 years now.

Good luck in your understanding of physics...

Alan?
Pmb?
Jeff?
Title: Re: Why do photons have the same speed regardless of the velocity of their source?
Post by: alancalverd on 22/03/2016 23:59:34
IMO...the Pound Rebka proves inconsistencies in currently held physics theory.
In everyone else's opinion, it shows that gravitational redshift can be measured and is exactly as predicted. But you are entitled to your opinion, which is much less boring.
Title: Re: Why do photons have the same speed regardless of the velocity of their source?
Post by: timey on 23/03/2016 00:47:33
IMO...the Pound Rebka proves inconsistencies in currently held physics theory.
In everyone else's opinion, it shows that gravitational redshift can be measured and is exactly as predicted. But you are entitled to your opinion, which is much less boring.

Alan - you are sooo droll!

Fortunately, being as you are a physicist, and an intelligent one in my non-boring opinion ;), you will be of the irk to push the boundaries of knowledge in the way that people of true thinking capacity are wont.

https://en.m.wikipedia.org/wiki/Doppler_effect

Quoted from link above:
"For waves which do not require a medium, such as light or gravity in general relativity, only the relative difference in velocity between the observer and the source needs to be considered."

Could you please answer these following questions?

The Doppler shift link above indicates that a Doppler shift cannot be achieved without some relative velocity occurring between observer and source.
Is this true?

The Pound Rebka consists of a man made Doppler shift measuring a naturally occurring Doppler shift within the gravitational shift of light.
Is this true?

The Pound Rebka, apart from the speaker vibration used to create the man made Doppler shift, involves no relative difference in velocity between the observer and the source, and with regards to the phenomenon of natural Doppler shift being measured by the man made Doppler shift, the gravitational field causing the gravitational shift in the light is not experiencing any change in its distance or in its strength of gravitational force other than the changes contained within the remit of the distance of the experiment.
Is this true?

Because if these things are true, then it doesn't matter if the experiment showed that Einstien predicted the gravitational shift correctly, the point of interest is that if a natural Doppler shift of light can occur without any relative motion between observer and source in a gravitational field, then this calls into question the fact of redshift being indicative of an expanding, accelerating universe, because the Pound Rebka has indicated, viewed from this perspective, that there was no expansion or contraction of space time within gravitational shift in light.
If there was no expansion or contraction of the distance, then what has caused the relative motion of the naturally occurring Doppler shift in the gravitationally shifted light?  Both the distance within the gravitational field and the speed of light are constant!
Title: Re: Why do photons have the same speed regardless of the velocity of their source?
Post by: JoeBrown on 23/03/2016 01:12:31
Okay, now I'm starting to understand your reasoning, tho I don't agree.

You've been terming all shift in frequency of light as doppler shift.  Even though doppler shift is only associated with differing in relative velocities.

It would have been clear if you stated that its "interesting that shift occurred in relatively static source/receiver".  But you did not, you called it doppler shift.

It is interesting that they found shift in light frequency to be equal and opposite in static relative motion in a vertical orientation, more so, because it is what evidence for gravitational shift they sought.

Would be less interesting if they also conducted the experiment in a horizontal orientation and produced the same results.  Then it wouldn't be thought to be the effect of gravitational shift or that it invalidates the principle and/or the experiment and wouldn't be considered evidence of gravitational shift.
Title: Re: Why do photons have the same speed regardless of the velocity of their source?
Post by: PmbNEP on 23/03/2016 04:12:56
Quote from: timey
Very good Pmb!!!

Now could you please tell me how, within the Pound Rebka experiment, a Doppler shift can be identified within the gravitational shift of light when there is no relative motion between the light source and the receiver other than the oscillations of the speaker cone...
Doppler shift is function of the speed of the source and nothing else. The source of gamma rays is mounted in the center of the loudspeaker and is attached to the cone so that as a signal is sent to the speaker  the source moves back and fourth. Therefore there is a relative motion between source and receiver. There can only be a Doppler shift when the source is moving as is the case here.

Note: This has nothing to do with the original question so this is an off-topic discussion.
Title: Re: Why do photons have the same speed regardless of the velocity of their source?
Post by: Colin2B on 23/03/2016 09:34:39
Note: This has nothing to do with the original question so this is an off-topic discussion.
It is also moving perilously close to a new theory discussion, which could be split out and moved.
It must also be confusing the socks off the question poster!
Title: Re: Why do photons have the same speed regardless of the velocity of their source?
Post by: timey on 23/03/2016 11:58:25
Doppler shift is function of the speed of the source and nothing else. The source of gamma rays is mounted in the center of the loudspeaker and is attached to the cone so that as a signal is sent to the speaker  the source moves back and fourth. Therefore there is a relative motion between source and receiver. There can only be a Doppler shift when the source is moving as is the case here.

Note: This has nothing to do with the original question so this is an off-topic discussion.

So Pmb - you are now telling me that the experimenters created a Doppler shift of gamma rays by a mechanism that is clearly obvious, and well documented in Pound Rebka link, (and indeed by most of the posters here, including myself)... and then used this man made Doppler shift to measured WHAT?
Because as far as I understand, they used this man made Doppler shift of the gamma ray, to measure a naturally occurring Doppler shift of light being shifted by the gravitational field!!!
True or false?

Although the man made Doppler shift was created by a slight variance in distance, caused by the speaker oscillations of 'back and forth', the naturally occurring phenomenon of gravitational shift that it was measuring occurred over a fixed distance of 22.5 metres.  There was no relative motion between the source and receiver with regards to the phenomenon being measured.
True or false?

With regards to the original posters question, you answered this question earlier this thread with the """correct""" answer already.  It's called invariance, and no one actually knows """how""" a photon travels at speed of light from a light source that is moving, it is only known what the light will be doing from an observers point of reference.

If you care to explore the parameters of the Pound Rebka experiment with me, as I am doing, I """can""" possibly actually answer the original posters question more precisely, because I think that perhaps I """do""" know how light moves across space in a gravitational field.

Btw...are you Pmb Pete who used to post before?  If so, I'm glad to see you back.

P.S.  Colin, if it's bothering the status quo, by all means split the thread and put it in New Theories.

P.S.S.  Joe - in answer to your post, please read the Wiki link on Doppler shift more carefully.  You will see that it is a documented fact that light undergoes a Doppler shift, that it is referred to as a Doppler shift, and is responsible for lights wavelength which is proportional to frequency.  The man made Doppler shift matched the frequencies of this naturally occurring phenomenon, cancelling out the gravitational shift of the gamma rays, more gamma rays were absorbed by the receiving sample and the number of gamma rays detected by the scintillation counter dropped accordingly, constituting the measurement, via the speaker 'phase', of the naturally occurring phenomenon of a Doppler shift in the gravitational shift of light.
Title: Re: Why do photons have the same speed regardless of the velocity of their source?
Post by: PmbNEP on 23/03/2016 13:06:21
Quote from: timey
So Pmb - you are now telling me that the experimenters created a Doppler shift of gamma rays by a mechanism that is clearly obvious, and well documented in Pound Rebka link, (and indeed by most of the posters here, including myself)... and then used this man made Doppler shift to measured WHAT?
Because as far as I understand, they used this man made Doppler shift of the gamma ray, to measure a naturally occurring Doppler shift of light being shifted by the gravitational field!!!
If this thread was about the Pound-Rebka experiment then I wouldn't have responded to this thread because I'm not an experimental physicist. As you said, everything you want to know about that experiment is posted in Wikipedia. I only responded to the question I quoted. I simply don't know why they had to have a time varying Doppler shift in the signal. I do, however, know experts who'd know this. I'll ask them about it and get back to you.
Title: Re: Why do photons have the same speed regardless of the velocity of their source?
Post by: JoeBrown on 23/03/2016 13:42:55
Note: This has nothing to do with the original question so this is an off-topic discussion.
It is also moving perilously close to a new theory discussion, which could be split out and moved.
It must also be confusing the socks off the question poster!

The original poster stated it was an enlightening argument with amusement, here:
quote author=NeT-HeaD link=topic=66096.msg483797#msg483797 date=1458670736

I've found the off topic premise concerning, confusing and aggravating.  Seems timey is enjoying itself inciting it.
Title: Re: Why do photons have the same speed regardless of the velocity of their source?
Post by: timey on 23/03/2016 14:42:37
I've found the off topic premise concerning, confusing and aggravating.  Seems timey is enjoying itself inciting it.

IMO Joe - 'you' are finding 'on topic' discussion confusing!

It was Colin who, quite appropriately to the topic I might add, brought up the subject of a Doppler shift in light in response to the original posters question.

My contribution has been in response to Colin's commentary!
Title: Re: Why do photons have the same speed regardless of the velocity of their source?
Post by: timey on 23/03/2016 14:49:04
If this thread was about the Pound-Rebka experiment then I wouldn't have responded to this thread because I'm not an experimental physicist. As you said, everything you want to know about that experiment is posted in Wikipedia. I only responded to the question I quoted. I simply don't know why they had to have a time varying Doppler shift in the signal. I do, however, know experts who'd know this. I'll ask them about it and get back to you.

Pmb - Thank you for your honesty... and, to say so, I'd be most grateful, and look forward to hearing back from you!
Title: Re: Why do photons have the same speed regardless of the velocity of their source?
Post by: Colin2B on 23/03/2016 23:27:39
I simply don't know why they had to have a time varying Doppler shift in the signal.
My understanding is that this was the easiest way of creating a variable speed of the source towards and away from the receiver., change the frequency, change the speed, change the Doppler shift. They found the point at which this Doppler shift canceled out the gravitational shift and the frequency gave them the speed, hence shift. Sorry to labour this, but explanation for those who have not read the detail.

Timey, I find your use of Doppler shift to describe gravitational shift as seen by observers in the gravitational field to be confusing in this thread. Gravitational shift is only seen as a Doppler shift by the free falling observer, ground observers attribute it to the acceleration of the photons increasing photon energy, hence frequency, but due to clock rate differences can only measure std speed of light. Again sorry to labour this, but I'm aware a wider audience is reading this.

Can we separate the terms for clarity.
Title: Re: Why do photons have the same speed regardless of the velocity of their source?
Post by: timey on 24/03/2016 00:15:52
Colin - To clear this up for any confused readers, the reason* why Pmb does not know why the Pound Rebka had to use a time varying Doppler shift in the signal, (this being the man made Doppler shift used to measure the natural phenomenon) is because the speed of light is constant, and over a constant distance there 'should not' be any such relative motion as to cause a Doppler shift (this being the naturally occurring phenomenon) in the gravitational shift in light over a fixed distance of the gravitational field.

As per GR and space time, redshift is indicative of an expansion of distance between light source and receiver, and blueshift is indicative of a contraction of distance between light source and receiver.

If anyone is confused by my observation here, really...don't worry, you have very good reason to be...

This being the point I am making!

*Pmb, if this is not the reason, I apologise for my presumptuous-ness, and invite you to correct me.
Title: Re: Why do photons have the same speed regardless of the velocity of their source?
Post by: alancalverd on 24/03/2016 06:36:30
The Pound Rebka consists of a man made Doppler shift measuring a naturally occurring Doppler shift within the gravitational shift of light.
Is this true?
No, no, no. It uses Doppler shift to measure gravitational shift. They are not the same thing, but both have the same effect on wavelength.

Please read this very carefully

Gravitational shift is NOT Doppler shift.

I say again

Gravitational shift is NOT Doppler shift

which is what everyone else has been saying and Messrs P & R understood from the outset.

Please do not talk about "gravitational Doppler shift" - there is no such thing. Gravitational shift is due entirely to the presence of a gravitational field. Doppler shift is due entirely to relative motion.

There is no confusion in anyone's mind but yours, and that only arises because you have invented a term that nobody else uses and has no meaning! 

Title: Re: Why do photons have the same speed regardless of the velocity of their source?
Post by: PmbNEP on 24/03/2016 07:14:17
Quote from: timey
Colin - To clear this up for any confused readers, the reason* why Pmb does not know why the Pound Rebka had to use a time varying Doppler shift in the signal, (this being the man made Doppler shift used to measure the natural phenomenon) is because the speed of light is constant, and over a constant distance there 'should not' be any such relative motion as to cause a Doppler shift (this being the naturally occurring phenomenon) in the gravitational shift in light over a fixed distance of the gravitational field.
That's not why I don't know why the experimenters used Doppler shift in there experiment. The reason is that I simply don't know the theory behind the workings of the experiment itself. For example; it may be that they got a decrease in experimental error by doing it that way. I simply don't know.

However there simply is no theoretical reason why they had to do it like that. Theoretically all they needed was a photon emitter and a photon detector which measures wavelength.

By the way it's not true that the speed of light is constant. That only holds in inertial frames, i.e. in gravitational fields. Here is a derivation for a uniform gravitational field/accelerating frame of reference and around a spherical body (i.e. Shwarzschild spacetime).

Go to www dot newenglandphysics dot org slash physics_world slash gr slash c_in_gfield.htm
Title: Re: Why do photons have the same speed regardless of the velocity of their source?
Post by: Colin2B on 24/03/2016 09:00:28
Theoretically all they needed was a photon emitter and a photon detector which measures wavelength.
When I read the experiment, this is why I assumed they used the loudspeaker. They didn't have a detector accurate enough to measure the shift, so they used the go/no go absorption to see when the Doppler shift matched the Grav shift, neat.

....because the speed of light is constant, and over a constant distance there 'should not' be any such relative motion as to cause a Doppler shift.
Ok, now I understand why you keep stressing constant distance. Ignoring the loudspeaker, between the top of the tower there is no motion to cause a Doppler shift - that's because there is no Doppler shift as Alan says, just a gravitational shift, which is not caused by movement.

As per GR and space time, redshift is indicative of an expansion of distance between light source and receiver, and blueshift is indicative of a contraction of distance between light source and receiver.
Only in the case of Doppler shift

If anyone is confused by my observation here, really...don't worry, you have very good reason to be...

This being the point I am making!
The confusion is clearing. I am beginning to understand why you are making your claims and where the errors are. Thanks for the clarification.

The following from Pmb explains.
By the way it's not true that the speed of light is constant. That only holds in inertial frames, i.e. in gravitational fields.
Remember also that as a consequence rate of clock is slower deeper in the Grav field, which matches the blue shift - this plus energy change, etc are all part and parcel of an interlocked group of  phenomena with the same cause, as described in GR.

Title: Re: Why do photons have the same speed regardless of the velocity of their source?
Post by: timey on 24/03/2016 11:15:14
Ok - in answer to the last 3 posts:

Alan, Pmb, Colin... good, good, now we have all understood that I am talking about a Doppler shift that the Pound Rebka identified in the gravitational shift, and that gravitational shift causes an 'expansion' and 'contraction' of wave'length'... Can you please tell me 'where' in the distance of 22.5 metres  the extra, or lesser length of the waves exists?

I understand that there is an aspect of time dilation that 'will' affect the journey of the light, however this time dilation is contrary to the direction of the contraction and expansion of the wavelength and 'cannot' explain the Doppler shift in the gravitational shift...
Title: Re: Why do photons have the same speed regardless of the velocity of their source?
Post by: puppypower on 24/03/2016 11:27:24
Consider, as an analogy for a photon, a boat traveling on the water. The boat is analogous to the particle, while the wake created by the boat is analogous to the wave aspect of the photon. If we had two tunnels under the bridge, the boat; particle, can only go through one tunnel at a time, while the wake, if spread out, can go through both tunnels.

The boat needs a constant supply of energy to keep moving at velocity V. This energy is needed to overcome friction between the boat and water. If the boat cuts the engines, the particle will slow, coast and then stop, while the wake will decay and finally stop. The speed of light is needed to maintain the wake/wave of the photon particles.

The question becomes how does a photon move through the medium of space-time and not slow down or speed up? What is the source of the propulsion energy?

If we assume the speed of light is the ground state of the universe, this implies all inertial references will be at higher potential. In this case, the constant speed of light would be connected to photons being constantly induced into the ground state ay C. The photons are constantly induced to higher potential; less than C by inertial. They cyclically drop back to the ground state; speed of light, while the energy differential is given off; motor that generates the wake/wave. 

One analogous way to look at this is connected to a house with a variety of electrical circuits, all using the same ground; earth. The ground is common to all the circuits and is at lower potential than any voltage/current configuration we may use. The potential difference between the inertial states, and the ground, is the energy potential used to drive the particle, which creates an analogous wake in space-time. This wake can become modified with the medium of space-time; red and blue shift, but the ground never changes, so the particles are always moving at C.

Fair enough PuppyPower... BUT... in Pound Rebka """Where"" in space time does this wave LENGTH exist if the distance between light source and receiver is held 'static', with respect to each other, on the basis that the oscillations of the speaker are creating a Doppler shift that cancels out the gravitational shift of the light?

The oscillations of a speaker, can be traced back to the energy signals being added to the speaker. If we shut off the power to the speaker there are no waves. We need a source of power. The energy input propels the speaker forward, which then flexes back to the rest position, due to the potential energy stored within the elasticity of the speaker, caused by the original forward energy that was added.

If we assume C is the ground state, the power circuit for all energy analogy speakers, is part inertial, which can be any of a variety of inertial states, static or in motion. It is also connected to the C ground state which is the same for all speakers. The speaker is driven forward by the potential energy returning to the C ground state. The return wave of the speaker is connected to the elasticity of the space-time medium speaker. If we took away the C ground state, the speaker would stop and no waves will appear. Energy moving at the speed of light, in all references, means the speaker is plugged in.

The Doppler shift of the speaker is driven in one direction. The rebound, in the other direction, is more reactive and is caused by the potential energy stored in the speaker, due to the forward energy pulse. The wave motion is due to the flow of energy from inertial back to the C ground.

Another visual is a water wheel, where the movement of water from a position of potential back to a ground state, results in the water wheel defining a wave. The water analogy for energy waves is time potential. The stationary water wheel defines a circle in space. However, it will define a wave in time.

(https://www.thenakedscientists.com/forum/proxy.php?request=http%3A%2F%2F1.bp.blogspot.com%2F-cBUd7QHWgK4%2FT1xNIB3CmfI%2FAAAAAAAALdE%2FMw22YcrwJ6E%2Fs1600%2FBreastshot%2Bwater-wheel.gif&hash=cccbec10f604796299ee1f836939b1a5)
Title: Re: Why do photons have the same speed regardless of the velocity of their source?
Post by: Colin2B on 24/03/2016 12:22:19
now we have all understood that I am talking about a Doppler shift that the Pound Rebka identified in the gravitational shift,
Ok, as long as you understand I'm not talking about a Doppler shift, because that's not what they identified, it doesn't exist.

and that gravitational shift causes an 'expansion' and 'contraction' of wave'length'... Can you please tell me 'where' in the distance of 22.5 metres  the extra, or lesser length of the waves exists?
They exist at every point in the 22m. The gravitational shift varies continuously from top to bottom so the frequency/wavelength also varies continuously, becoming increasingly blue shifted down the tower. You can view this as increasing energy as the photons accelerate, or you can look at the slowing of clocks - because each observer will measure the speed of light as constant, the frequency will be seen as increasing (shorter wavelength).

I understand that there is an aspect of time dilation that 'will' affect the journey of the light, however this time dilation is contrary to the direction of the contraction and expansion of the wavelength and 'cannot' explain the Doppler shift in the gravitational shift...

It's not a Doppler shift.
The time dilation - slower clock - explains the blue shift as I said above, how it is contrary?. The time dilation is what 'causes' the acceleration, it's all interlinked just as in SR - time dilation and length contraction are aspects of the same effect, you can't add both together and get an additional effect.
Title: Re: Why do photons have the same speed regardless of the velocity of their source?
Post by: timey on 24/03/2016 12:51:26
Colin - For the man made Doppler shift to cancel out the blueshift and redshift frequencies, the man made Doppler shift must match those frequencies exactly!  You are wrong.  There is a Doppler shift in the gravitational shift of light.  The Pound Rebka proved it!  The wiki Doppler shift link states it!

If the time dilation aspect was responsible for relative motion within the gravitational shift of light, then you would see 'longer' wave lengths in blueshift, and 'shorter' wave lengths in redshift.

Furthermore:

https://en.m.wikipedia.org/wiki/Doppler_effect

Quoted from above link:
"For waves that propagate in a medium, such as sound waves, the velocity of the observer and of the source are relative to the medium in which the waves are transmitted. The total Doppler effect may therefore result from motion of the source, motion of the observer, or motion of the medium. Each of these effects is analyzed separately. For waves which do not require a medium, such as light or gravity in general relativity, only the relative difference in velocity between the observer and the source needs to be considered."

I repeat: "only the relative difference in velocity between the observer and source needs to be considered."

The Pound Rebka measured a Doppler shift in the gravitational shift over a distance of 22.5 metres.  There was no relative difference of velocity between light source and receiver with respect to the gravitational shift they measured over a distance of 22.5 metres.

Now I'm sorry, but physics cannot have its cake and eat it.  Either there 'must' be relative velocity between light source and receiver that expands or contracts the distance between them, in which a wavelength experiences changes in distance that affords the wavelength's expansion or contraction, or there must be a 'medium' in the gravitational field that explains the relative motion of a Doppler shift in the gravitational shift of light!
Title: Re: Why do photons have the same speed regardless of the velocity of their source?
Post by: Colin2B on 24/03/2016 17:18:37
Colin - For the man made Doppler shift to cancel out the blueshift and redshift frequencies, the man made Doppler shift must match those frequencies exactly! 
Agreed

You are wrong. 
Thanks

There is a Doppler shift in the gravitational shift of light. 
No. as I've said before there are 2 different effects at work here

The Pound Rebka proved it! 
No it didn't

The wiki Doppler shift link states it!
Where

If the time dilation aspect was responsible for relative motion within the gravitational shift of light, then you would see 'longer' wave lengths in blueshift, and 'shorter' wave lengths in redshift.
The time dilation isn't responsible for any relative motion (because there isn't any relative motion) but it does affect the frequency.
Lets take an eg with simple numbers:
Observer high in weak gravity field measures time of 1 cycle as being 1s = 1Hz.
Further down the field clocks run slow (time dilation) so the same cycle could be measured as say 0.5s =2Hz.
This is an increase in frequency - blueshift - and a shortening of wavelength.
No movement needed, no doppler shift, just due to grav field and clocks running slow.

Furthermore:

https://en.m.wikipedia.org/wiki/Doppler_effect

Quoted from above link:
"For waves that propagate in a medium, such as sound waves, the velocity of the observer and of the source are relative to the medium in which the waves are transmitted. The total Doppler effect may therefore result from motion of the source, motion of the observer, or motion of the medium. Each of these effects is analyzed separately. For waves which do not require a medium, such as light or gravity in general relativity, only the relative difference in velocity between the observer and the source needs to be considered."

I repeat: "only the relative difference in velocity between the observer and source needs to be considered."
Although that is true it only applies to the doppler part of the experiment.
The presence of a medium is irrelevant, this is light not sound.

The Pound Rebka measured a Doppler shift in the gravitational shift over a distance of 22.5 metres. 
No, they used a doppler shift to measure the gravitational shift.

As I said before, between the top of the tower and the bottom there is a gravitational shift which they wanted to measure. As Pmb says, they could have used an accurate freq counter, but I don't think they had one accurate enough. So, my reading of the experiment is that they looked for a way of creating a null. Using a null reading is extremely accurate and commonly used eg an interference pattern or a bridge cct. In this case they decided to create the null by making a doppler shift using the movement of the loudspeaker, I suspect the other end was a simple scintilation counter. When the doppler shift equals the grav shift they cancel and the counter shows null. The value of the doppler shift = the value of the grav shift and they can work out the doppler value from the frequency fed into the loudspeaker.
At no point are they measuring a doppler shift in the grav shift, just using one to measure the other.

There was no relative difference of velocity between light source and receiver with respect to the gravitational shift they measured over a distance of 22.5 metres.
They didn't try to measure this.
Because of time dilation the observer at top of tower and the one at the bottom will measure the same speed of light.

Now I'm sorry, but physics cannot have its cake and eat it.  Either there 'must' be relative velocity between light source and receiver that expands or contracts the distance between them, in which a wavelength experiences changes in distance that affords the wavelength's expansion or contraction, or there must be a 'medium' in the gravitational field that explains the relative motion of a Doppler shift in the gravitational shift of light!
Physics isn't trying to have it's cake and eat it. You are looking for something that isn't there.

Anyway, that's it from me as I suspect you aren't going to believe me. Either Pmb or Alan will no doubt put me right as I've been winging it here.
Title: Re: Why do photons have the same speed regardless of the velocity of their source?
Post by: timey on 24/03/2016 18:00:32
Colin - I cannot 'do' these multi quote posts on my phone easily at-all, and my phone is all I have at mo...so please excuse me, but can we do this one or two portions at a time?

It states in the Doppler shift link that Both light waves and gravity waves are subject to Doppler shift.  In lights case this is because wavelength is proportional to frequency.  It is the extra, or lesser length in the wavelength that is """synonymous""" to a Doppler shift.

Now - if the man made Doppler shift is cancelling out the natural frequency of the gravitationally shifted light, then perhaps the man made Doppler shift is creating a time variance in accordance with the time drift found in the gravitational field.  If it were then my question is:
Where in the fixed distance of 22.5 metres do the extra or lesser lengths of these wavelengths exist?

However, on the basis that a person living on top floor in a high rise, will, over the course of 79 years, only be 1 minute older than his downstairs neighbour on the ground floor, (or something similarly within this tiny region anyway), I think we can pretty much rule out the fact of the speaker cone creating such a 'subtle' vibration at between 10 and 50 hertz.  The vibration that the speaker vibrated at is more synonymous with a much greater factor of Doppler shift than this.  One that matches the frequency being proportional to the wavelength, I'd imagine.  So again, my question is:
Where in the fixed distance of 22.5 metres do the extra or lesser lengths of these wavelengths exist?
And:
What is causing the relative motion, that the man made signal cancels out,  in the gravitational field?

Btw, Colin, I'm not winging it.  I've had training in sound engineering and have been home studying physics for 7 years.  I appreciate that this does not mean that I'm correct in what I'm observing, I may well be mistaken, but I've racked my brain with it all for many years now, and really, I don't actually think I am mistaken ;)
Title: Re: Why do photons have the same speed regardless of the velocity of their source?
Post by: timey on 24/03/2016 19:01:04
https://en.m.wikipedia.org/wiki/Relativistic_Doppler_effect
Title: Re: Why do photons have the same speed regardless of the velocity of their source?
Post by: Colin2B on 24/03/2016 23:25:44
Ok, we have some common ground. I worked as a sound/vision engineer (BBC trained) and studied Applied Physics day release/evenings until I went to study full time. When I say winging it, I mean reading the various reports of Pound Rebka experiment to make sense of it, most of it is fairly clear but you need a bit of guess work to fill in the gaps.

So, let's concentrate on one question at a time as you suggest.

"Can you please tell me 'where' in the distance of 22.5 metres  the extra, or lesser length of the waves exists?"

They exist at every point in the 22.5m. The gravitational shift varies continuously from top to bottom so the frequency/wavelength also varies continuously, becoming increasingly blue shifted down the tower. You can view this as increasing energy as the photons accelerate, or you can look at the slowing of clocks - because each observer will measure the speed of light as constant, the frequency will be seen as increasing (shorter wavelength). Remember these are nm wavelengths.

"What is causing the relative motion, that the man made signal cancels out,  in the gravitational field?"
That's easy, there is no relative motion, because the gravitational shift is not due to motion.

I don't know why you have quoted the 2 Wiki articles because neither of them suggest that Doppler is involved in gravitational shift, nor that PR found it as a cause of the shift.
Title: Re: Why do photons have the same speed regardless of the velocity of their source?
Post by: timey on 25/03/2016 00:18:52
Damian and I did a lot of work for the BBC, back in the day, from Korner studios, Scrubs Lane, when I was in training as his tape hop.  Jingles, radio recordings, etc.  It wasn't just that we were so close to the White City BBC studios, Damian had the best live room ever!  I remember you posting that you are into sound... doing outdoor recordings as I recall, you play guitar as well if I remember rightly?

Ok - so, yes, of course the wavelengths exist at every point in the distance of 22.5 metres.  Now this is where it gets interesting!  Because there is no relative motion caused by changes in the distance in the phenomenon being measured, only a fixed distance of 22.5 metres, we can now 'peg' gravitational shift in light, exactly to a gravitational field, which provides us with an absolute reference frame.  The reason why GR cannot provide us with such, is because a relativistic Doppler shift in light is thought to be synonymous with expansion or contraction of distance.  The PR shows that this is not the case.  There is a relativistic Doppler shift 'effect' within a distance that is not expanding or contracting.

And:
I said - "What is causing the relative motion, that the man made signal cancels out,  in the gravitational field?"
You said "That's easy, there is no relative motion, because the gravitational shift is not due to motion."

Gravitational shift in light is due to an expansion, or contraction, of distance between light source and receiver.  The relative velocity between light source and receiver expands or contracts the gravitational field.  Hence redshift being indicative of an expanding universe.

In the PR, light travels a fixed distance of 22.5 metres, at the speed of light.  A time variance has been added to the test signal via the vibration of the speaker. This constitutes relative motion.  The signal matches the frequency of the gravitational shift.  Does this not indicate that the relative motion of the signal is reflected in the gravitational shift?

Of course, it could be that all these considerations are contained and addressed within the maths of the PR, as Pmb suggests.  I wouldn't have a clue being a non-mathematician, but the written in 'words' dialogue doesn't state this as being the case though, and usually it would...

P.S.  Not sure why you are asking that about the Wiki links.  The Doppler shift link mentions light and gravity.  The relativistic Doppler 'effect' link explains frequency and wavelength, and drops it back to the Lorentz transformations which I have addressed in my "4 a deeper discussion - is distance an absolute invariant" thread.  Are you sure I'm not just confusing you and the issue by my 'shoddy' use of terminology ;) ?
Title: Re: Why do photons have the same speed regardless of the velocity of their source?
Post by: JoeBrown on 25/03/2016 02:29:42
Couple quick observation about your argument timey:

Pound Rebka did two observations without motion.

Of the 2 still (speaker off or absent) observations one was photons going up and the other with photons down.

The results showed equal but opposite shift:
 shift toward red (photon going up)
 shift toward blue (photon going down)

Now if this presumed gravity shift has anything to do with cosmological shift, why would the results be the opposite change in shift?

Another problem with cosmological shift and the experiment is that the effect of cosmological shifts are so subtle the same shift would require distances of well over a million light years (which is quite a bit further) instead of 22.5 meters.

But Like you said, I don't get it, so I really shouldn't bother chiming in.  Perhaps *I* should go read it again.  ;)
Title: Re: Why do photons have the same speed regardless of the velocity of their source?
Post by: timey on 25/03/2016 03:07:09
Ok Joe - I now formally apologise to you for saying you have short legs.  That which seems obvious to me is clearly not as obvious to others as I had thought it might be!

Yes of course they measured both directions.  Yes redshift is light going away from Earth and blueshift towards the earth, as expected.  It is the 'time drift' that occurs, according to established physics, in the opposite direction.  Faster time away from the earth, and slower time towards the earth.

I personally do not see any difference between redshift occurring near a gravity field or in deep space, it's only that the wavelength will be longer in the weaker gravity field.

You do bring up a valid point about 'distance' though, this being the point I'm making.  If relativistic Doppler 'effect' can happen in a fixed distance of 22.5 metres, the extra, or lesser wavelength can, (in some way), be pegged directly to the gravitational field.  This would rather mean that estimated cosmological distances based on redshift and parallax method are then rendered questionable though!

I 'can' tell from your commentary that you are still not quite getting some of the basics, but so long as you aren't pointing "Doh's" at 'me' over it, I can forgive you... ;)
Title: Re: Why do photons have the same speed regardless of the velocity of their source?
Post by: JoeBrown on 25/03/2016 04:11:06
I don't believe cosmological shift is directly related to distance.  To my knowledge, I'm in the minority, in this regard.

Its why I vehemently objected to gravity and doppler being termed the same.  They both relate to GR but are achieved by adjusting different variables.  My thesis depends on this.

While I don't wish my thesis to be proved false, I have yet to determine a method to prove it otherwise.

shameless retort replaced with shameless plug (: please review my hypothesis :)
http://www.thenakedscientists.com/forum/index.php?topic=66046.0
Title: Re: Why do photons have the same speed regardless of the velocity of their source?
Post by: evan_au on 25/03/2016 04:43:52
Quote from: puppypower
The Doppler shift of the speaker is driven in one direction. The rebound, in the other direction, is more reactive and is caused by the potential energy stored in the speaker, due to the forward energy pulse.
The most convenient way of driving a loudspeaker is with a sine wave, which has both negative and positive voltage excursions.

The loudspeaker does not passively fall back to the center position, but is actively accelerated through the center position, and out the other side.

So you can't talk about this experimentin terms of a "ground state" of the loudspeaker, or a "ground state" of the speed of light.

Quote from: Colin2B
rate of clock is slower deeper in the Grav field, which matches the blue shift
There may be a bit of confusion in various posts of the thread about whether the gravitational shift is a red shift or a blue shift.

As JoeBrown mentioned, they actually tried the experiment in both directions (up and down). As I read it:

Mössbauer spectroscopy
The Pound–Rebka experiment made use of Mössbauer spectroscopy (http://en.wikipedia.org/wiki/M%C3%B6ssbauer_spectroscopy#Typical_method). I presume that this technique is normally performed in a horizontal direction, so that gravitational shift does not affect the results.

With 14keV gamma rays from Fe57, you need to have a relative velocity of about 10.6mm/s for maximum absorption, due to factors like the internal magnetic field of an iron nucleus (around 30 Tesla).

The additional gravitational shift due to a tower of 22.5 m is very small - the velocity only differs by 7.5x10-4 mm/s. No wonder they had to run the experiment in both directions, to double their signal-to-noise ratio.
Title: Re: Why do photons have the same speed regardless of the velocity of their source?
Post by: evan_au on 25/03/2016 05:07:19
A Question: Why weren't the results of this experiment totally swamped by thermal vibrations of the iron atoms?

Wikipedia (http://en.wikipedia.org/wiki/Atom_vibrations) suggests that
Quote
Elastic waves of different lengths, frequencies, and amplitudes run through crystalline solids at all times. The typical order of the atomic vibrations frequencies is 1013 Hz, and that of the amplitudes is 10−11 m.

We could express one of these thermal vibrations as a distance displacement s=10-14*sin(2π*1013t) mm
Differentiating, this suggests a velocity of the iron atoms of around v=(2π/10) * cos(2π*1013t) mm/s, which has a peak velocity of around 0.6 mm/s.

How could they detect a difference of only 7.5x10-4 mm/s when the thermal noise floor is about a thousand times larger?

There is a picture of the apparatus here, but I don't see a way of cryogenically cooling the emitter or detector.
https://books.google.com.au/books?id=_X5nbOSHuAMC&pg=PA180
Title: Re: Why do photons have the same speed regardless of the velocity of their source?
Post by: Colin2B on 25/03/2016 08:43:57
Quote from: Colin2B
rate of clock is slower deeper in the Grav field, which matches the blue shift
There may be a bit of confusion in various posts of the thread about whether the gravitational shift is a red shift or a blue shift.

As JoeBrown mentioned, they actually tried the experiment in both directions (up and down). As I read it:
  • with the emitter at the base of the tower, and the detector at the top: In this case, the emitter (loudspeaker) must be moving upwards (towards the detector) for the gamma rays to be absorbed by the detector; the blue shift due to the Doppler effect exactly cancels the gravitational red shift.
  • with the detector at the base of the tower, and the emitter at the top: In this case, the emitter (loudspeaker) must be moving upwards (away from the the detector) for the gamma rays to be absorbed by the detector; the red shift due to the Doppler effect exactly cancels the gravitational blue shift.
It was the 2nd case I was refering to, moving from weaker field at the top of the tower giving a blue shift. I was trying to show Timey, with the simplified eg I gave, that the time dilation between top and bottom of the tower can explain a blue shift, whereas she says it should give a redshift, and a faster rate of clock at the tower bottom would be necessary for a blue shift. I havent seen her calculation for this. (quote from Timey -"If the time dilation aspect was responsible for relative motion within the gravitational shift of light, then you would see 'longer' wave lengths in blueshift, and 'shorter' wave lengths in redshift.")



In the PR, light travels a fixed distance of 22.5 metres, at the speed of light.  A time variance has been added to the test signal via the vibration of the speaker. This constitutes relative motion.  The signal matches the frequency of the gravitational shift.  Does this not indicate that the relative motion of the signal is reflected in the gravitational shift?
No, there is no evidence that the gravitational shift contains any relative motion or doppler shift. This would be like saying a gravitational wave contains an interference pattern because that was used to measure it.


Not sure why you are asking that about the Wiki links.  The Doppler shift link mentions light and gravity.  The relativistic Doppler 'effect' link explains frequency and wavelength,
I mentioned them because you provided links to them and said "There is a Doppler shift in the gravitational shift of light.  The Pound Rebka proved it!  The wiki Doppler shift link states it!"
I have read both these links and although they do, as you say, mention light and gravity, nowhere does it state that there is a Doppler shift in the gravitational shift of light or that Pound Rebka proved it!  Indeed it provides  a link to gravitaional time dilation which describes the effect almost exactly as I did.

I don't feel I am confused by your terminolgy, as you seem to consistently link frequency shift to being a doppler effect only.
But I am confused as to how you calculate that time dilation should result in longer wavelengths. I've shown above how it doesn't and can do no more.
Title: Re: Why do photons have the same speed regardless of the velocity of their source?
Post by: puppypower on 25/03/2016 11:08:41
Red and blue shift of energy due to the Doppler shift can also be explain with the concept of entropy. If entropy is defined as increased disorder, two references moving apart at velocity V, will increase entropy. Since an increase of entropy needs to absorb energy, the observed red shift reflects the movement toward lower potential energy quanta to satisfy the needs of the higher entropy; energy conservation.

When two references approach, they are moving toward a state of higher order and therefore lower entropy. The loss of entropy will give off energy potential, which is reflected in the blue shift, where photons appear to gain the energy potential. This is also due to energy conservation.

If we expand a gas, all the gas molecules will see higher entropy with the gas cooling; red shift in the IR. If we compress the gas, the entropy is reduced, energy is given off and the gas gets warmer; blue shifted in IR. If we focus on only two molecules either coming together or moving apart; doppler shift explanation, we will still get blue and red shift in the heat signature; IR.
Title: Re: Why do photons have the same speed regardless of the velocity of their source?
Post by: agyejy on 25/03/2016 12:00:35
Red and blue shift of energy due to the Doppler shift can also be explain with the concept of entropy. If entropy is defined as increased disorder, two references moving apart at velocity V, will increase entropy. Since an increase of entropy needs to absorb energy, the observed red shift reflects the movement toward lower potential energy quanta to satisfy the needs of the higher entropy; energy conservation.

When two references approach, they are moving toward a state of higher order and therefore lower entropy. The loss of entropy will give off energy potential, which is reflected in the blue shift, where photons appear to gain the energy potential. This is also due to energy conservation.

If we expand a gas, all the gas molecules will see higher entropy with the gas cooling; red shift in the IR. If we compress the gas, the entropy is reduced, energy is given off and the gas gets warmer; blue shifted in IR. If we focus on only two molecules either coming together or moving apart; doppler shift explanation, we will still get blue and red shift in the heat signature; IR.

I don't think you really understand entropy all that well. Also you seem to be under the impression that a gas always cools when expanding and heats up when compressed. This is obviously not the case or we wouldn't have things like isothermal expansion or isothermal compression which are specifically expansion and compression at a constant temperature.
Title: Re: Why do photons have the same speed regardless of the velocity of their source?
Post by: PmbNEP on 25/03/2016 12:54:33
Quote from: timey
Btw...are you Pmb Pete who used to post before?  If so, I'm glad to see you back.
Yep. That's me. I was unable to figure out how to login with my old account so I had to create a new one. Thanks for the warm welcome back.

Again, please take note that I simply don't know why they use Doppler shift. And I only trust textbooks and journal articles for thinks like that.
Title: Re: Why do photons have the same speed regardless of the velocity of their source?
Post by: timey on 25/03/2016 13:05:50
Evan - (Colin this also applies to you). I have no confusion over the direction of the time drift, or that redshift makes for longer waves travelling from bottom to top of tower, and blueshift shorter waves from top of tower to bottom.

You said:
""with the emitter at the base of the tower, and the detector at the top: In this case, the emitter (loudspeaker) must be moving upwards (towards the detector) for the gamma rays to be absorbed by the detector; the blue shift due to the Doppler effect exactly cancels the gravitational red shift.""

To be clear, it was the gamma rays that were emitted from the speaker and traveled upwards, the speaker did not move, apart from the movement of the fabric of the cone caused by vibration.

""for the gamma rays to be absorbed by the detector; the blue shift due to the Doppler effect exactly cancels the gravitational red shift.""

A man made Doppler shift was created via the vibration of the speaker constituting, as Pmb suggested, a time variance in the signal that exactly matched the Doppler 'effect'... of... what did you say there Evan?  That it cancels out the gravitational redshift?  There was no contrary signal of gamma rays being emitted from bottom of tower when they conducted the experiment from the top of tower, only the receiver was at the bottom of the tower at the time.  What the signal travelling from top of tower cancelled out was the Doppler 'effect' in the gravitational shift when the test signal exactly matched this naturally occurring Doppler 'effect'.  This being the relativistic Doppler 'effect' that light experiences in a gravitational field.
Same for the redshifted gamma rays from bottom of tower to top.
Yes - within the calculations, the top of tower and bottom of tower results were correlated with each other.

(Edit:  Evan - having re-read what you said here again, I find that you have indeed explained the situation correctly...although perhaps my commentary clears up any possible misinterpretation of moving speakers and contrarily 'emitted' signals...anyway, I apologise, because your commentary is correct.)

My point being that a relativistic Doppler 'effect' of the gravitational shift in light is NOT supposed to occur, unless the observer is moving towards the light source, or away from the light source, or the light source is moving away from the observer, or towards the observer, or both light source and receiver are moving relative to each other with a difference in relative velocity.
The Pound Rebka obtained the results of a relativistic Doppler 'effect' in the gravitational shift over a fixed distance of 22.5 meters!

My point about the time drift was, that IF the time drift were to have caused any relative motion within this fixed distance, as to then be the cause of 'relative motion' within the fixed distance of 22.5 metres, then as the time drift is contrary to the direction of the expansion and contraction of the waves, ie: redshift occurs travelling into a weaker gravity field, but in a weaker gravity field time runs faster, this - IF we were looking for 'relative motion' within the fixed distance, which of course a 'time variance' would provide us with - would in fact cause redshifted wavelength to become shorter.  ie: that it should take a shorter amount of time for the light to cover the same unit of distance.
The time variance in the time drift is also far too subtle to be replicated by the vibration of the speaker cone that is creating a time variance in the test signal.
Therefore we can rule out the factor of the time drift being responsible for the 'matching time variance' of the test signal found within the naturally occurring phenomenon being measured.

That the Doppler shift, time variated test signal matched the relativistic Doppler 'effect' of gravitational shift over a fixed distance constitutes an illogicality within the remit of GR.  Where, within the fixed distance of 22.5 metres, in space time are the extra or lesser lengths of wavelength existing.  Either distances expand or contract as per the Lorentz transformations, or they don't!!!
Within the Pound Rebka it would appear that they don't!!!

What is causing the 'relative motion' in light over this fixed distance???

(Colin - the fact that my concept of an inverted time dilation, for massless entities, within the remit of my Inverted Time Theory, exactly explains this 'time variance' matched in the naturally occurring phenomenon by the test signal, is of course what is drawing me to the premiss of this experiment)

P.S.  Pete - good, good!  As far as I'm concerned the forum has been feeling a little empty without your contributions.  I know we've had our little run in's, ;), but your knowledge and clarity of explanation is without question.
Title: Re: Why do photons have the same speed regardless of the velocity of their source?
Post by: alancalverd on 27/03/2016 11:06:44

To be clear, it was the gamma rays that were emitted from the speaker and traveled upwards, the speaker did not move, apart from the movement of the fabric of the cone caused by vibration.


The cone fabric is irelevant. The gamma source was attached to the speaker core, so the source moved.

And since no part of the speaker can move faster than the speed of sound, the relativistic Doppler effect is irrelevant too - the result can be calculated adequately from the classical Doppler equations.

Keep it simple, chaps!
Title: Re: Why do photons have the same speed regardless of the velocity of their source?
Post by: timey on 27/03/2016 13:28:10
Yes - but it is not 'sound' that is causing there to be a time variance of the emission of the gamma rays within the distance of 22.5 metres between the speaker and the receiver.  It is the motion of the speaker cone that the gamma ray emitter is mounted upon that is doing this.  It's the back and forth vibration that moves the light source back and forth, not the sound itself.

The point being though, Alan, is that a natural relativistic Doppler 'effect' found within a fixed distance matched this time variated signal exactly!

Therefore:
Do distances expand and contract via the Lorentz transformations or not?
Title: Re: Why do photons have the same speed regardless of the velocity of their source?
Post by: evan_au on 27/03/2016 23:27:48
Quote from: Timey
Do distances expand and contract via the Lorentz transformations or not?
Yes, distances are affected noticeably at relativistic speeds (eg velocity > c/10).

But the speeds in this experiment were around 11mm/s, ie velocity < c/109, which is nowhere near fast enough for relativistic Doppler effects to be visible. The non-relativistic Doppler effect will be sufficient to interpret the effect of the loudspeaker in the Pound-Rebka experiment.

So this non-relativistic Doppler shift was enough to cancel the relativistic Gravitational shift.

Quote
Doppler 'effect' of the gravitational shift in light
- Gravitational shift is a frequency shift.
- Doppler shift is a frequency shift.
- But Gravitational shift is not a Doppler shift.
- This description is similar to the logic which says "1 is a number; 2 is a number; therefore 1=2."
 
Title: Re: Why do photons have the same speed regardless of the velocity of their source?
Post by: alancalverd on 28/03/2016 00:03:38
The point being though, Alan, is that a natural relativistic Doppler 'effect' found within a fixed distance matched this time variated signal exactly!
No, a thousand times no.

Gravitational redshift is NOT Doppler shift.

For the last time before I lose all patience with you, they measured gravitational shift  by comparing it with a known Doppler shift which they produced by moving the detector at a known speed. That's all.
Title: Re: Why do photons have the same speed regardless of the velocity of their source?
Post by: alancalverd on 28/03/2016 00:11:57
A Question: Why weren't the results of this experiment totally swamped by thermal vibrations of the iron atoms?
Because thermal vibration is random. It's fairly easy to extract a signal of known phase and frequency from random noise.
Title: Re: Why do photons have the same speed regardless of the velocity of their source?
Post by: timey on 28/03/2016 00:37:20
Evan - good, your description is very concise!

So... Where within the gravitational shift measured is the relative motion of the test signal replicated?  In other words, where in space time are the extra, or lesser lengths of wave located?

If they are located within the fixed distance of 22.5 metres, this means that either the speed of light is not constant over the naturally occurring variance of the gravitational shift, or there is a time variance within the gravitational shift matching the time variated test signal... (The gravitational shift being the cause of the experiments observed redshift, blueshift results, not the test signal.)

Alan - yes... Agreed!!!  And I don't see why you keep banging on about the fact.  I'm saying that a Doppler shift was used to identify something within a fixed distance of gravitational shift that 'resembled' a Doppler shift.  It had to 'resemble' a Doppler shift because a Doppler shift was used to measure it... ???
Title: Re: Why do photons have the same speed regardless of the velocity of their source?
Post by: alancalverd on 28/03/2016 00:48:27
If they are located within the fixed distance of 22.5 metres, this means that either the speed of light is not constant over the naturally occurring variance of the gravitational shift, or there is a time variance within the gravitational shift matching the time variated test signal

Or that photons demonstrate a blueshift on moving through a gravitational gradient, which is what everyone else thinks.   

I'm not a great proponent of consensus in science, but this does seem to be the simplest description of what happens.
Title: Re: Why do photons have the same speed regardless of the velocity of their source?
Post by: timey on 28/03/2016 01:15:43
Well yes of course everyone thinks that Alan, including me... but are you not at all curious as to how photons do this?
Does it not raise any questioning in you as to how photons can do this within a fixed distance of gradient in the gravitational field?
Surely over a fixed distance there would be no need to add a time variance to the test signal in order to measure the gravitational shift...why would it not work by just emitting the gamma rays from a static position?
Title: Re: Why do photons have the same speed regardless of the velocity of their source?
Post by: alancalverd on 28/03/2016 09:42:20
Nobody added a "time variance".

The Mossbauer effect has an amazingly narrow bandwidth. You can demonstrate nuclear resonant fluorescence if the source and detector are fixed in a horizontal plane, but the effect disappears if either moves at more than about 1 cm/second with respect to the other. This is a classical resonance phenomenon involving conservation of energy and momentum, nothing to do with Doppler or gravitation. 

Now when the P-R experimental plane was vertical, the incoming signal was red or blue shifted so could not produce resonance in a stationary receiver - they had to move the source or receiver to compensate for the gravitational shift.

One advantage of using a clean sinusoidal movement such as a loudspeaker coil is that you can phase-lock your detector to the source motion and thus infer the speed at which the source is moving when the detector resonates. Remember that if displacement = sin ωt, then velocity = cos ωt at any given time t.

www.tau.ac.il/~lab3/MOSSBAUER/Literature/Mossbauer_Harvard.pdf describes a standard teaching experiment and is well worth reading.
Title: Re: Why do photons have the same speed regardless of the velocity of their source?
Post by: evan_au on 28/03/2016 10:07:28
Quote from: Timey
where in space time are the extra, or lesser lengths of wave located?
You have to observe the experiment from some frame of reference - preferably one which is static relative to the surface of the Earth (or your own acceleration will further confuse the discussion).

Probably the most useful frame of reference is adjacent to the detector at the top of the tower, with the emitter at the bottom of the tower.
Gamma Rays emitted near the highest point of the speaker cone travel many wavelengths less distance than gamma rays emitted near the lowest point of the speaker cone. But the effect of gravitational shift, which happens over 22.5m dwarfs the effect of the speaker moment (which is probably 0.01m). 

(I hope this helps - just reverse the logic if you want to consider gamma rays emitted at the top and absorbed at the base of the tower.)

Quote
why would it not work by just emitting the gamma rays from a static position?
There is a very narrow window of frequencies/wavelengths that an Fe57 nucleus will emit or absorb. You have to get the frequency just right (to within ≈1011), or it will go straight through the detector plate and hit the scintillation counter behind it.

The nucleus which emits the gamma ray is in a metal matrix with other metal atoms, but there is still a microscopic recoil which loses some energy from the photon. There are other effects in the emitting and absorbing nucleus that means you need to have the emitter approaching the detector at 10.6258 mm/second to be absorbed. And that is if you are doing the experiment in a horizontal orientation.

Repeating the experiment in a vertical orientation produces an additional tiny gravitational frequency shift, which is what Pound & Rebka were trying to measure.

See: https://en.wikipedia.org/wiki/M%C3%B6ssbauer_spectroscopy#57Fe_M.C3.B6ssbauer_spectroscopy
Title: Re: Why do photons have the same speed regardless of the velocity of their source?
Post by: puppypower on 28/03/2016 12:08:42
Photons have two aspects. On the one hand, they travel at the speed of light which is the same in all references. Photons also express  wavelength and frequency, which are dependent on inertial reference. A moving source can impact wavelength/frequency, since the inertial reference changes, but it does not impact the speed of light. These are two different legs of the photon.

One leg of the photon is always in the ground state of the speed of light, with this ground state the same for all inertial references. The other leg is in inertial reference and will therefore change with reference. If we were to travel at the speed of light, we would not be able to see the all the variety of wavelength, since space-time will be contracted to a point-instant. Wavelength and frequency differentiation is an artifact of inertial reference and space-time.

Because of the two legs, a potential is set between the ground state at C, which does not see any variety in terms of wavelength and frequency; unbiased reference for all, and the wavelength and frequency, determined by space-time and inertial reference. A tug of war between the two; inertial and the C-ground state, results in the wave nature of photons. If we alter inertial reference, the parameters of the tug of war change, thereby altering the wavelength and frequency seen in inertial reference. The ground state never changes.

The red shift of the universe reflects that the C ground state wins all the tug of wars over time. The energy value of photons lose potential to the ground state and red shift.

As an analogy say we have a series of different weights hanging from a series of standard springs. Gravity defines the ground state common for all the springs; how they will hang at steady state. If we start at the top of all the springs and let them all fall, they will each vibrate; wavelength and frequency, based on the potential between the common ground state and the weight/tension in each spring.

Say we had one static spring/weight and we conserve the energy from one vibrating spring and transfer this energy to the static spring. The frequency will change base on its weight. Weight itself is dependent on the ground state of gravity.
Title: Re: Why do photons have the same speed regardless of the velocity of their source?
Post by: jeffreyH on 28/03/2016 14:26:19
The more important question is how the shift in wavelength fits in with Lorentz transformations, time dilation and length contraction. Is there an inequality in the effects on particles with rest mass and massless bosons. This is obviously true simply because of the speed of light. Is this hiding an important inequality?
Title: Re: Why do photons have the same speed regardless of the velocity of their source?
Post by: jeffreyH on 28/03/2016 14:32:26
Near the event horizon of a black while fermions are doomed photons can still happily escape. This is where the hidden inequality will be most apparent.
Title: Re: Why do photons have the same speed regardless of the velocity of their source?
Post by: timey on 28/03/2016 15:29:21
The more important question is how the shift in wavelength fits in with Lorentz transformations, time dilation and length contraction. Is there an inequality in the effects on particles with rest mass and massless bosons. This is obviously true simply because of the speed of light. Is this hiding an important inequality?

Jeff - clearly you are firing on all cylinders... :)

At two moments in each cycle of the audio feeding into the speaker, while the speaker is moving upwards at about 10.6258 mm/second, the gamma rays will have just the right frequency to be absorbed by Fe57 in the detector. For these moments, the output of the scintillation counter drops, because more of the gamma rays are being absorbed.
At these moments, the Doppler shift and gravitational shift cancel, and you get the same result as if you conducted the experiment horizontally.

Gamma Rays emitted near the highest point of the speaker cone travel many wavelengths less distance than gamma rays emitted near the lowest point of the speaker cone. But the effect of gravitational shift, which happens over 22.5m dwarfs the effect of the speaker moment (which is probably 0.01m). 

Yes - there is a variance in the distance between the back and forth motion of the speaker cone.  The photon that is emitted from the speaker cone from the forward position of the cone, takes a shorter amount of 'time' to travel to the receiver than the photon that is emitted from the backward position of the speaker cone.  This constitutes a 'time' variance in the test signal.

How does a time variance of this magnitude exist within the gravitational shift over 22.5 metres?
Title: Re: Why do photons have the same speed regardless of the velocity of their source?
Post by: alancalverd on 28/03/2016 16:17:10
The photon that is emitted from the speaker cone from the forward position of the cone, takes a shorter amount of 'time' to travel to the receiver than the photon that is emitted from the backward position of the speaker cone.
But remember that velocity is the time derivative of position. The cone isn't moving at either the forward or backward position, so there is no Doppler shift.
Title: Re: Why do photons have the same speed regardless of the velocity of their source?
Post by: timey on 28/03/2016 16:58:10
Alan - The info that Evan quoted stated that the gravitational shift was cancelled out when exactly the correct 'back and forth' motion of the speaker was matched by the correct position of 1 photon being emitted at 1 distance, and 1 photon being emitted at a different distance... (It didn't quite say this, but this 'is' what it meant, because frequency is the result of oscillation)

I don't understand your fixation with the terminology Doppler shift... :(
The experiment states that a Doppler shift was created to measure the gravitational shift.  What more do you want me to say about it?

My observation is that 'relative motion' was used in the test signal to measure gravitational shift over a 'fixed' distance.  The relative motion was caused by, what looks very much to me, like a 'time delay'.  The light emitter was being reverberated.

(Edit:  The noise from the speaker will be having no effect on the light whatsoever - it is the noise that drives the speaker, and hence the subsequent oscillations in the speaker cone, upon which the light emitter is mounted, that is creating the frequency in the test signal that cancels out the gravitational shift.)
Title: Re: Why do photons have the same speed regardless of the velocity of their source?
Post by: alancalverd on 28/03/2016 23:24:18
Alan - The info that Evan quoted stated that the gravitational shift was cancelled out when exactly the correct 'back and forth' motion of the speaker was matched by the correct position of 1 photon being emitted at 1 distance, and 1 photon being emitted at a different distance... (It didn't quite say this, but this 'is' what it meant, because frequency is the result of oscillation)


It didn't say that, because it didn't mean that. Doppler shift derives from velocity, not position.

Quote
I don't understand your fixation with the terminology Doppler shift... :(
The experiment states that a Doppler shift was created to measure the gravitational shift.  What more do you want me to say about it?


Preferably nothing, once you accept the difference between Doppler shift, gravitational shift, and frequency mixing. Which, judging by your next remark, you haven't

Quote
My observation is that 'relative motion' was used in the test signal to measure gravitational shift over a 'fixed' distance.  The relative motion was caused by, what looks very much to me, like a 'time delay'.  The light emitter was being reverberated.

(Edit:  The noise from the speaker will be having no effect on the light whatsoever - it is the noise that drives the speaker, and hence the subsequent oscillations in the speaker cone, upon which the light emitter is mounted, that is creating the frequency in the test signal that cancels out the gravitational shift.)
Title: Re: Why do photons have the same speed regardless of the velocity of their source?
Post by: timey on 28/03/2016 23:40:59
Alan... I do know the difference between a Doppler shift, gravitational shift and frequency.  I know the difference between lights frequency, and sounds frequency and how the frequency of both affect wavelength.

However, from everything that I have read about concerning space time and the Lorentz transformations, 'relative motion' should not occur for light within a fixed distance of the gravitational field.

Whatever it is you wish to call it, relative motion was created in the test signal.  This 'relative motion' was reflected in the phenomenon being measured over a fixed distance.

Why?
Title: Re: Why do photons have the same speed regardless of the velocity of their source?
Post by: timey on 31/03/2016 15:03:47
...and helicopters were not deployed?

(Perhaps it's a ridiculous question...)
Title: Re: Why do photons have the same speed regardless of the velocity of their source?
Post by: timey on 31/03/2016 16:25:43
...and helicopters were not deployed?
(A 'Natural Born Killers' reference, btw.)

Sound is the propagation of a pressure wave through a medium of particles.

Redshift and blueshift is the propagation of a light wave through a medium of gravitational shift.

Maybe it is a ridiculous question, but, I repeat...  In the Pound Rebka, whatever it is you wish to call it, 'relative motion' was created in the test signal.  This 'relative motion' was reflected in the phenomenon being measured over a 'fixed' distance.

Why?
Title: Re: Why do photons have the same speed regardless of the velocity of their source?
Post by: alancalverd on 03/04/2016 23:27:51
Because the "phenomenon measured over a fixed distance" was a frequency shift of the incoming photon due to the gravitational gradient over that distance. Here's the A level explanation (the real experiment involves a tiny bit more sophistication)

Static, "classical" case: 57Fe nucleus emits a photon with frequency f → photon detected by the receiver if and only if the received frequency is exactly f. This is the Mossbauer effect.

Static, relativistic case: Add a gravitational gradient: 57Fe nucleus emits a photon with frequency f → gravitational red (or blue) shift means the receiver sees  f + δf ∴ no signal.

Now add a Doppler shift by moving the receiver.  At some point during the sinusoidal motion  the Doppler shift will be -δf so the receiver will see the incoming photon as having frequency f and you will get a signal out of the receiver.

We know that the Doppler shift is given by v/c where c is the speed of light and v is the instantaneous velocity of the receiver, so we can calculate δf, the gravitational shift.

What makes this a really clever experiment is that you are measuring a tiny change in a really big number, with some remarkably crude equipment (a bass loudspeaker!). Hats off (and indeed a Nobel prize) to Rudolf Mossbauer, massive praise to Pound and Rebka, and everlasting respect to all  bass-playing physicists.

Database Error

Please try again. If you come back to this error screen, report the error to an administrator.
Back